Você está na página 1de 119

1.

Two firms - Marksman Marketing Services Private Ltd


and Vectracom were in the news recently. Why?

Ans: The PCB had granted the exclusive rights to


disseminate information relating to scores via SMS on
wireless and mobile telephones on a global basis to
VectraCom Private Limited. Marksman Marketing Services
Private Ltd had obtained exclusive territorial rights
for India from the VectraCom.

2. Kyon Ki, Home Delivery, Neal N Nikki, Kisna and


James are vying for top honours at this awards. Name
this newly launched awards and the publication behind
the awards.

Ans: The Outlook Follywood Film Awards (Outlook


Magazine).

3. Its been included as an Olympic sport 12 times


since 1964.The FIL based in Germany is its governing
body. Georg Hackl of Germany has the rare distinction
of having a podium finish in 5 consecutive Olympics.
Who represents India in this Olympic discipline?

Ans: Shiva Keshavan. The discipline is Luge.

4. Easy One. Connect Ratan Tata - DIN 0001.

Ans: DIN is the Director Identification Number. Ratan


Tata is the first executive in corporate india to get
a director identification number. This is under a
surveillance system introduced by the government to
keep an eye on the activites of the directors of
companies.

5. The Kadima (meaning Forward) Party formed in the


year 2005 is the ruling party of which country? (Its
founder has been in the news recently).

Ans: Israel (Founded by Ariel Sharon).

6. CALTECH, FOX, RAFFEYSOD and GO NAVY are some of the


many alterations that were made to this famous
landmark. Which one?

Ans: The HOLLYWOOD sign.

7. The Essar Group and the Bajaj Group are the Indian
partners of these companies. Both these companies have
been hogging attention in the Sports pages of the
Indian subcontinent. Name the companies.

Ans: Hutchison and Allianz. (Sponsors of the ODI and


Test series between India and Pakistan).

8. Which film production company's logo takes its


inspiration from the Mahabharata and features Arjun at
the Draupadi swayamvar taking aim at the fish?

Ans: Rakeysh Omprakash Mehra Productions (Rang De


Basanti).

9. The Elder Pharma Group (license holders in India of


the famous Tiger Balm) is the main sponsors of the
Filmfare Awards this year. Which TV personality is the
Director of the company?

Ans: Anuj Saxena (Abhay of KKusum and now Jatin of


Kumkum).

10. Derek type Question - In which Indian daily would


you find Blue, Red, Yellow and Black dots in each of
the pages that appear in color.

Ans: Well, its Cyan, Magenta,Yellow and Black (thanks


to Rohan Nair). The daily or dailies are those brought
out by the Times of India Group. The colors are also
visible on the ENIL (Entertainment Network India Ltd.
that runs Radio Mirchi).

11. Overdrive is a monthly auto magazine brought out


by the Infomedia India Limited. A few months ago it
tied up with a more than 140 year old publication in
the UK to launch which magazine in India?

Ans: Cricinfo.

12.The President of this Latin American country


recently cut his salary by more than half leading to a
review of the salaries of all public sector employees
in the country as no official can earn more than the
President. Which country?

Ans: Bolivia (President: Evo Morales).

1. Topical Q - Name the venture cap company founded by Raj Kondur and Ashish
Dhawan :: Chrysalis

2. Fill in the blank for book - Life as I have known it has been _ _ _ (3 words) by
Colonel Harland Sanders :: Finger lickin good

3. Info Edge owns jeevansaathi.com and 99acres.com. Which is the flagship site? ::
Naukri.com

4. Who hosts the talk show "La Noche del 10 "? Famous sportsman :: Diego
Maradona
5. Who is the author of God's Debris? :: Scott Adams

6. What's the word/phrase that refers to the placement of an advertisement


surrounded entirely by editorial or program content. :: Island Advertisement

7. You gotta know BPO and KPO. So whats CRO thats becoming big biz in south India
specially? :: Clinical Research

8. What specific product is described/branded as Sexy Rexy, Playgirl, Disco Dancer,


Sweet Caroline etc...:: Roses

9. Ticker symbol of which company is ITWO? :: i2 tech

10. Who topped the list of non-corporate brands in India, a study by


businessweek/interbrands (Sachin, Ash, Bangalore were 2,3,4 I think) :: Taj Mahal

11. Don't remember :: NA

13. "Not Allowed To Cry" book, author Ingrid Albuquerque-Solomon - The untold
story of how India's most honored and accomplished industrial family challenged
creation’s most dreaded disease Cancer – and won!. Which industrialist :: Aditya
Birla

14. Which company is behind


Saksham, meaning 'self reliant', is an attempt to offer 50000 rural kiosks in six states
to create a strong IT rural ecosystem :: Microsoft

16. How do we better know 'Charles Townsend Detective Agency's key employees? ::
Charlie's angels

17. Which company has lunched kid ware by the name Zapp? :: Raymonds

18. Org chart of this big Auto company show jaguar, Mazda etc...:: FORD Motors

19. Visual - Google doodle of Martin Luther King Jr


20. Visual Ad - Showing lots of nuts on the ground and one upright bolt in the
middle :: AXE

21. Visual Ad - showing Air India's maharaja announcing Cochin Airport's


internationalization operations launch :: Air India

22. Audio - open up ...:: Nescafe

23. Visual of tintin character - tell the name and his profession :: Jolyon Wagg,
Insurance agent

24. TV Ad of Killer jeans (worn out jeans)

25. TV Ad LG DIOS - inventive thinking

1. which brand get its name from an abbreviation of synthetic phenol ?

2. "a brilliant sun " is a documentary movie made about which famous industrialist in india
?

3. what is common to a billionaire S.I newhouse Jr.vogue, vanity fair, details, lucky and
self ?

4. name the editior in chief of times of india and hindustan times ?

5. whats common to adolf hitler and a silk route famous number (song) ?

6. which pharma company owns brands like dcold, krack cream and borosoft ?

7. name that american president who was appointed without elections ?

8. "perot sysytems" a software firm was under whose control before they turn out to be an
independent company ? (TELL THE COMPANY)

9. name first indian model (miss india) to have competed for miss world tytle ?

10. which famous indian couple from bollywood is deprived of any children till now ?
(need both names )

11. name that personality frm bollywood who has a perfume named after him and has
also won dada saheb phalke award ?

12. name the first woman frm raj kapoor family to have acted in bollywood movies ? ( i
mean first lady frm great kapoor family.. )
All questions are Google friendly and googling is perfectly legal and welcome. As one of my friend
said to me when I confessed about googling.

"When you Google, you learn" )

1. For all economists here - What are giffen goods?

2. Marketing world : Kotler gave 4 Ps. Who gave 4 As?

3. There used to be mailing list which was called Dakghar (Post house/office). Members wanted
to write about lot of Indian things, their opinions and share with others. With some tech help from
couple of members, it developed into a website and is doing very well. The logo of the site is
some carnation of yin and yang. Which website? The Hindi name translates into "good writings".

4. As per the Golf Digest magazine - who's the best golf player among Fortune 500 company
CEOs/CXOs?

5. Which company/ group in India has partnered CNN to come up with a tv series "Eye on India"

6. If we take billboard numbers (songs) in USA, which brand is mentioned most frequently in the
top 100 of this list in the year 2005?

7. When coming up with company name for this start up, one of the founder's wife thought the
word means new in French !! Ex-CEO of this firm now heads Google. Which company?

1. The business is based in Street, Somerset. Started


in 1825, by Quaker brothers Cyrus and James, as a
business making sheepskin rugs and slippers, using
out-workers in the village, it has expanded to become
a global shoe brand. Most famous for introduction of
“Hygienic Line”- shoes that fit the natural shape and
length of the human foot. Also the official supplier
of shoes to Britain's royal family.

Clarks Shoes, UK's largest brand, has tied up with


lifestyle retail company Lifestyle Asia in India.
Clarks is the official supplier of shoes to Britain's
royal family

2. “If there had been a capitalist down there at Kitty


Hawk, he should have shot down Orville and saved us a
lot of money." Who said this and why?
Warren Buffet. Buffet's Berkshire Hathaway holding
company lost a bundle investing in US Air

3. “The insight for this campaign is ‘disbelief’. When


someone tells us something that is too good to be
true, we often don’t believe him. Our first instinct
is to have a bet with him on it. So, we had this whole
theme of one guy believing in ___________ and the
other disbelieving him. In a way, all the commercials
so far are factual as well as entertaining, thus
coming under the genre of infotainment.”
What is being described?
SBI's Bets campaign

4. “In our Hindi ads, we have made use of Paresh


Rawal’s voice. However, for the south, we plan to use
the Tamil film director Visu’s voice in the ad”.
Which is the ad being described here?
Wipro Sanjeevani
Dominos was the most common Answer

5. Who owns the company 'Red Chillies’? Name the super


hit movie produced?
SRK and Gauri.The company produced Main Hoon Na
/Paheli

6. Aadhar is primarily a rural format selling fmcg


products, fertilizers, animal feed etc ideally
bringing the real rural masses into their stores while
Nature's Basket would be Urban format setup in major
capital cities. Whose initiatives are these?
Godrej

7. Easy one: Identify the ad:

Brushless or lather boys,


Whichever you've a notion
Fifty cents for the ____,
A dollar for the ______.
"______ means quality,"
Said the Captain to the Boson.
So look for the package with
The ship that sails the ocean.
Old spice

8. Which is the second most traded commodity in the


world, trailing only petroleum?
Coffee

1. Which book starts with these lines " I want to die at a hundred
years old with an American flag on my back and the star of Texas on
my helmet, after screaming down an Alpine descent ….. "? There are
enough clues in the questions.

It's Not about the bike - Lance Armstrong (seven-time Tour de France
champion cyclist)
2. One for the car enthusiasts what is the concept behind the
design of WOW car of Honda? Also expand WOW?

Honda Motor Co. has designed a car that's friendly for dogs — part of
the Japanese automaker's ongoing effort to create vehicles that are
easy to use and comfortable to ride in.

The W.O.W. Concept, which stands for "wonderful openhearted


wagon," is an exhibition model with no plans for commercial sale that
will be exhibited at the Tokyo auto show later this month.A special
crate for dogs in the glove apartment allows owners to interact with
their pets while driving. A bigger crate pops up from the floor in the
back seat area and can be folded back into the floor when it's not
needed. For even bigger dogs, just buckle them up with a special seat
belt to the floor.

3. Who said this "It's not like this is a village where you have to
have been here for 100 years." Clue – CEO of one of the big Asian MNC.

Sir Howard Stringer. Since being named Sony's first non-Japanese CEO
back in March, Stringer, 63, has thrown himself into the daunting task
of reviving the struggling $65-billion-a-year global electronics and
entertainment giant.

4. X and Y have come together to launch an air ambulance service


for critically ill patients named `Air Rescue One'. 'Air Rescue One'
aircraft will have a medical rescue team and equipment on board to
treat any eventuality not only on-the-spot but also on way to hospital.
Identify X & Y?

Escorts Heart Institute and Deccan Aviation

5. This Kimberly-Clark brand (X) has become a `genericized


trademark.' In 1930s it was advertised with a slogan "Don't Carry A Cold
In Your Pocket" this helped X product to replace product `Y'. Recently
Greenpeace. In 2005, Greenpeace launched the Kleercut campaign against
Kimberly-Clark to protest its methods of production of `X' and its
alleged use of ancient forests to produce disposable products. Identify
X?

Kleenex

6. Recently the FDA has approved Exubera. This drug is supposed


to change the way people with diabetes are treated. You just have to
tell me what is so special about Exubera?
Nearly 21 million Americans suffer from diabetes and the majority of
those with type 2 diabetes have uncontrolled blood sugar levels.
However, many people who need insulin, a proven effective treatment,
are fearful to inject it. Now, the FDA has approved Exubera, the first
inhaled and non-injectable insulin in the U.S. that could change the
way people with diabetes are treated

. Arnold Schwarzenegger, Sylvester Stallone, Burt Reynolds, Richard


Gere - all said no. Finally, who said yes?

BRUCE WILLIS, DIE HARD, THE MOVIE THAT TURNED JUST ANOTHER "MOONLIGHTING"
ACTOR INTO ONE OF THE TOP 5 HIGHEST PAID ACTORS IN HOLLYWOOD

BRUCE WILLIS IS FAMOUS FOR HIS "SAVE THE WORLD" KIND OF ROLES, WHICH HE
PARODIED IN THE FILM REFERED TO IN QUESTION 2

2. He quotes Nietzsche in the film "What doesn't kill you, makes you
stronger" He believes that destruction actually creates productivity,
a theory disproven by Frederic Bastiat in the "Parable of the Broken
Window". Name the character, the actor and film?

JEAN-BAPTISTE EMMANUEL ZORG, THE "QUINTESSENTIAL" MEGALOMANIAC FROM THE


FIFTH ELEMENT, BRILLIANTLY PORTRAYED BY GARY OLDMAN.

A VERY FAMOUS BRITISH ACTOR OF THE OLD CLASSICAL SCHOOL PLAYED THE
PRIEST, VITO CORNELIUS. (VERY FAMILIAR NAME, EH)

3. Which actor has played characters like Himmler, Dr. Joseph


Goebbels, Pontius Pilate and Bilbo Baggins?

IAN HOLM

IAN HOLM'S MOST FAMOUS PERFORMANCE BEFORE BILBO BAGGINS IN THE LORD OF
THE RINGS WAS AS AN ATHLETIC COACH

4. The title for which significant movie came to the scriptwriter as


an inspiration while watching BBC's religious music programme "Songs
of Praise" featuring the hymn "Jerusalem" by William Blake

CHARIOTS OF FIRE

THE LINES (AS PER MY MEMORY) GOES


BRING ME MY BOW OF BURNING GOLD
BRING ME MY ARROWS OF DESIRE
BRING ME MY SPEAR, O CLOUDS UNFOLD
BRING ME MY CHARIOT OF FIRE

THE THEME MUSIC OF CHARIOTS OF FIRE HAS ACHIEVED CULT STATUS AND ANY
SPORTS PROGRAMME ON DD USED TO FEATURE IT

5. Complete the name: _______ Odysseus Papathanassiou

EVANGELOS OR VANGELIS WAS THE MUSIC COMPOSER FOR CHARIOTS OF FIRE.


AMONG HIS OTHER FILMS WAS THIS CLASSIC RIDLEY SCOTT FILM FEATURING
HARRISON FORD AND RUTGER HAUER

6. While writing a screenplay, what phrase did William Burroughs coin


to call "a person who sells illegal surgical instruments"?

BLADE RUNNER

LOOKING AT RIDLEY SCOTT'S FILMOGRAPHY, YOU SEE A NUMBER OF FILMS


INCLUDING

7. ___________ Yvonne Dickinson & ________ Elizabeth Sawyer. Complete.

THELMA AND LOUISE, ONE OF THOSE RACY CHASE MOVIES

8. Scott Glenn in The Silence of the Lambs, Dennis Farina in


Manhunter, who in Red Dragon?

HARVEY KEITEL (JACK CRAWFORD) HAS HAD A NUMBER OF MEMORABLE CHARACTER


ROLES INCLUDING INVESTIGATOR HAL SLOCUMB IN THELMA AND LOUISE, MR WHITE IN
RESERVOIR DOGS, AND MANY MORE. THE MOST MEMORABLE ROLE (IMHO) WAS "THE
WOLF"

9. Three taglines one film, (I don't see how they can all talk of the
same movie. Too deep for me, but nice lines anyway). Just give the film

"Girls like me don't make invitations like this to just anyone!"


"You won't know the facts until you've seen the fiction."
"Just because you are a character doesn't mean you have character."

PULP FICTION
10. Finally, too much of working with banks, this question seems to be
a fallout of all that. What is the expiry date of Batman's Credit Card
in the film "Batman & Robin"

1. Recently Christina Aguilera allowed a UK company to use her name in their


label in a multi million pound contract. Which company ?

Ans : Basic Box

2. IOC/s branded petrol is called ‘Xtrapremium’, BPCL’s ‘Speed’, HPCL’s ‘Power’.


What is IBP’s branded petrol called ?
Ans : Josh. Now they are selling Xtrapremium.

3. This MD of a Bangalore based international company is a hardcore art critic. He


organized trips to Egypt and the ancient silk route of Indian artists and coaxed
them to arts inspired by the two great civilization of the ancient times. Name him.
Ans : Rakesh Agarwal, MD of LANXESS ABS ( earlier Bayer ABS )

4. ‘Blitz’ is the German slang for the logo of which brand of car ?
Ans : Opel

5. ‘Think Again’ is the tagline of which tv channel ?


Ans : National Geographic

6. ‘Clean Slate Consulting’ is the consulting outfit of which management guru ?


Ans : Narayan Pant ( Dean of INSEAD’s asian campus in Singapore )

7. Which brand of Volkswagen car so far sold maximum number ?


Ans : Volkswagen ‘Golf’, 23 million ( Volkswagen ‘Beetle’ sold 21.5 million )

8. Which is London’s first Indian Restaurant ?


Ans : Hindoostane Coffee House in Central London

9. What is the most famous brand of US based ‘Limited brands Inc’ ?


Ans : Victoria’s Secret

10. Which Indian company owns ‘Homefield International’ ?


Ans : Tata Chemicals

. What is your name??? (Be cool!!! Just to copy-paste)


Answers may vary ;)
1. Initially it was planned to release this movie in both English and Hindi but
due to some constrains the English version "Paint it yellow" didn't hit the
theatres. How do we know this movie better?
Rang de Basanti (universally cracked…)

2. "You can either agree with me or may be wrong"


"Well behaved women are rarely successful"
"I'm cute?"
"Don't get in my way".
Where would you have seen this?
Sania Mirza's T-shirts

3. Put fundae: 20-6, 8-10, 28-11, 6-2.


Quite vague!!!
Only one person came very close to the answer and so I have decide to
carry forward this question to COCKTAIL-VI with some additional funda so
that everyone of you may crack it…

4. Connect: East Timor, Ecuador, Marshall Islands, Federal states of


Micronesia, Republic of Palau and El Salvador (HINT: something financial!!!
Hope the list is exhaustive …)
Countries which have US dollar as their currency
BUZZ….. Poorest countries,
countries No taxes,
taxes bill gates earns more in a year than
these countries!!
countries etc etc…

5. Sitter. This organization was the first Japanese company to be listed on the
NYSE (New York Stock Exchange) .The company was established in 1946 by
founder Morita Akio as Tokyo Tsushin Kogyo Kabushiki Kaisha. Name the
world famous company.
Sony

6. X followed Pompey to Egypt where he restored Queen Cleopatra—earlier


deposed by her brother Ptolemy XIII—to the Egyptian throne. He soon brought
her to Rome as his mistress. X routed the rebellious king of Pontus in Asia
Minor, a battle in which the historian Suetonius quoted X as having made the
famous statement: "____ _____ _____"
Identify X and what was the statement made by him.

Julies Caesar- Vini, Vidi, Vici…


I came, I saw, I conquered…
Many got caeser but said "Et tu Brutus…"

7. Jay Leno introduced X in 2002 as "one in a billion". X has been described as


"one to watch out for" in an ongoing event. Identity X who is the lone
contestant for India in the ongoing event.
The lone Indian Luge contestant at Torino Winter Olympics Shiva
Keshavan
Also I suppose there are some more participants from India in other
events…

8. HP TRIVIA: Harry: Hungarian Horntail


Fleur: Welsh Green
Krum: Chinese Fireball
Cedric: ______ ________
Swedish Short snout
A lot of people said Norwegian Ridgeback…
The most sensible comment I got for this was "stop Harry potter" …

9. Think logically!!! . Who murdered one fourth of the world's population?


Cain…when he killed Abel (one among the first four survivors…)
Someone said dude you have missed the serpent…LOL!!!
BUZZZ….. (Most fauvorite answer-Hitler, wars, some disease, plague,
Tsunami!!!)

10. Born on January 8, 1935, X attended L.C. Humes High School and after
graduation worked as a truck driver in the city. In 1953, on his lunch break, he
recorded two songs for his mother at the Memphis Recording Service. These
songs brought him a recording contract and launched his rise to fame. The great
X is also not spared and mocked by Eminem in his famous album "Without
me". Identify X.
Elvis Presley

1. This was created by Rowan Atkinson, Richard Curtis and Robin Driscoll
with Rowan playing the lead and started on 1st January 1990 . An electrical
engineer with degrees from Newcastle and oxford universities, how do we
know Rowan Atkinson better?

ANS: Mr. Bean. I have also awarded points for Col. Black adder as spotted by
one quizzer. Even 2 or 3 chums said he is better known as Rowan Atkinson
2. HP TRIVIA: Parvati patil and Padma Patil are the two characters of Indian
origin in the Harry potter series. According to J. K. Rowling which book served
as an inspiration for these two characters?

ANS: Midnight's children by Salman Rushdie


Crazy: "Ans: Salman Rushdie's wife's name is Padma Lakshmi....so lets say she
got inspired from Salman Rushdie's The Ground Beneath Her Feet??!!"
, Sonia Gandhi…

3. This term entered usage as early as 1858 during a cricket match between an
All England team and Hallam XI at Sheffield's Hyde Park ground. H.H.
Stephenson performed something and a collection was made to buy him
something. But it took some two decades to get dignified in print to describe
something by Fredrick Spofforoth. Identify that something.

ANS: hat trick


After he achieved the feat collections were made to buy him a hat and thus
arrived the word…

4. Connect: Clippit, The dot, F1, Merlin, Mother Nature, Links, and Rocky
Microsoft office helpers
CHECK: click help > show the office assistant; default clippit appears…
Then right click on the pic and then click choose assistant…
U can see the animated pics of all the above mentioned…

5. In the movie Titanic the nude portrait of Kate winslet (Rose Dewitt Bukater)
was sketched by X. Also in the screen it was X's hand shown drawing and not
De Caprio's as we visualize. Identify X.
James Cameroon
Crazy answer by someone: "me"

6. Who was supposed to have asked this riddle to enter the city of Thebes
"What is it that has four feet in the morning, two at noon, and three at night?"
and those who didn't answer were killed. When Oedipus who was made the
king for solved the riddle by answering, "Man, who crawls on four limbs as a
baby, walks upright on two as an adult, and walks with the aid of a stick in old
age," she killed herself. Identify this mythological character, a semblance of
this is still famous.
ANS: sphinx
Thanks ankur
" u see a sphinx in HP book 4 where a sphinx asks a question to HP in the maze
during the triwizard tournament"

7. The face of the main character of the film was modeled after Albert Einstein,
Carl Sandberg and a pup dog. Michael Jackson recorded the theme song
"Someone in the dark", though it was not used, it features in the storybook
version of the film. When the cassette was released in the US, it was made of
green plastic. Identify the movie.

Extra terrestrial (E.T)

8. This word was coined in 1937 when the American food manufacturers Geo.
A. Hormel & Co (Hormel Foods Corporation) were looking for a name for their
new product- cans of meat. A cash prize of $100 was offered for the best
suggestion. What word did the winner suggest?
S-P-A-M

9. Thomas Alva Edison served as member of the board of directors of this


company till 1894. This MNC has wings in more than 100 and head quarters at
Fairfield, Connecticut.
Name the company.
GE (general elec)

10. Half a league, half a league,


Half a league onward,
All in the valley of Death
Rode the ___ _________.
'Forward, the Light Brigade!
Charge for the guns!' he said:
Into the valley of Death
Rode the ___ _______.

Fill the blank, also name the poem.

Six hundred and the poem is "The charge of the light brigade" by Lord
Tennyson
Sadhashiv says…
"a very moving poem about the best fighting unit having to 'cross the rubicon'
in its literary sense all because the generals did not give them the order to
stop...very very charging..."Canon to the right of them, Cannon to the left of
them....."

vivek said..
"I am wondering why you gave six hundred as the blank instead of giving the
blank in the place of Light Brigade. That would have made more this question
more interesting".

Yellow, Pink, Orange, Pineapple

Match these with a state Government. Business connection of course.

The answer is SIKKIM. The names are those of their different state lottery schemes

1. What is common to the muslim names Ar-Razzaq, Al-Mazid, As-Sami, Az-Zahir ?

Ans :

2. Sean Connery charged $250000 for his less than two minutes role in this film. He donated the
amount for charity. Which film ?

Ans :

3. The screen name of which famous bollywood actress was given by Dilip Kumar ?

Ans :

4. 'Cora' is the wife of which famous European sportsman ?

Ans :

5. Which animal has same number of neckbones as that of human ?

Ans :

6. Which was the first genetically modified vegetable that was allowed by US Federal Govt ?

Ans :
7. 'Abhaji' was the middle name of this Bollywood actor. Name him.

Ans :

8. 'Punkster' is the pet dog of which Bollywood actress ?

Ans :

9. Who is the missing one in the list based on a particular performance ? N Astle, ? , H Gibbs, A
Gilchrist, I Botham.

Ans :

10. 'Uncommon Sense' .... adline of which corporate ?

Ans :

1. In the Green Hills of Africa, Ernest Hemmingway


placed this novel in historical context saying “ All
Modern American literature comes from one book called
_______, But it’s the best book we had. All American
writing comes from that. There was nothing before.
There has been nothing as good since “. Which novel
was he referring to?
THE ADVENTURES OF HUCKLEBERRY FINN, MARK TWAIN

2. Along with Truman Capote, he is considered a


pioneer of Creative non fiction genre. Born to a
Jewish family, he was bought up in Brooklyn, New York
and studied Aeronautical Engg in Harvard in 1939.
During WW2, he served in South Pacific. Based on his
personal experiences during the War he wrote a novel,
which was hailed as one of the greatest American
novels. He was awarded the Pulitzer for his non
fiction work on anti Vietnam war protests called
Armies of the Night. He has also written biographies
of Marilyn Monroe, Pablo Picasso and Lee Harvey
Oswald. Name this author.
NORMAN MAILER

3. Derived from a German word meaning “novel of


education”, it refers to a novel which traces the
spiritual, moral and psychological growth of a
character from childhood to maturity. David
Copperfield, To Kill a Mockingbird, Harry Potter
series are examples of this sort of novel. What genre
is this called?
BILDUNGSROMAN
4. Considered one of the greatest novels of all times,
it deals with Philip Carey who suffers from a club
foot.It traces the hero’s journey’s through London,
Germany and Paris, and his destructive relationship
with a Cockney waitress Mildred. Which novel?
OF HUMAN BONDAGE

5. He lives at No 7, Saville Row, Burlington Gardens,


London. He is a member of the Reform Club. His main
past time is reading newspapers and playing a game of
whist. He fires his servant James Forster, as he has
bought the shaving water, four degrees below normal.
Name this eccentric fictional character.
PHILEAS FOGG

6. Complete the quote : “Some say that Signor


Bonicini, Compared to Handel’s a mere ninny; Others
aver to him that Handel, is scarcely fit to hold a
candle, Strange! That such high dispute should be
‘Twixt _____________”
TWEEDLEDUM AND TWEEDLEDEE

7. Apart from his famous novels, this author was also


one of the early writers of travel books. Based on his
travels in France, he wrote Travels with a Donkey in
Cevennes in 1879, and this was one of the first books
to promote hiking and camping.
R.L.STEVENSON

8. This 1992 Merchant Ivory production starring Helena


Bonham Carter and Emma Thompson, was based on this
E.M.Forster novel dealing with life in England at the
beginning of the 20th century. It deals with 3
families Wilcoxes- rich aristocrats, the middle class
Schlegel sisters and the working class of the Basts.
HOWARD'S END

9. This novel gets it’s name from the following line


in Julius Caesar :” Cry havoc and let slip
____________”. It is about the story of a group of
mercenary soldiers who are hired by a British
industrialist to overthrow the government of a
fictional African country called Zangaro.
DOGS OF WAR

10. He used to stay in a trailer with his wife


Tabitha. He wrote short stories to make ends meet. One
of his first ideas was a girl with psychic powers,
however he grew discouraged and threw the novel into a
trash can. His wife encouraged him to write it, and he
later sent it to Doubleday. This novel would prove to
be a cult favorite and later become a bestseller and
also a successful movie. Name the author and the book.
STEPHEN KING,CARRIE
Hello All,

A new edition of the quiz for you.

1. Which company is the sole seller of Segway’s electric human transporter ?


Ans :

2. Dharmendra is the mastermind behind which Mumbai restaurants ?

Ans :

3. 'East-Watch' is the newsletter of which organisation ?

Ans :

4. 'Baniya' community of India gets its name as a consequence of what ?

Ans :

5. Which shirt brand has come out with a shirt exclusively for lefthanders ?

Ans :

6. Which Indian company is named after the name of its founder and the village from
which he hails from ?

Ans :

7. This popular beer brand is named after the month when it is traditionally brewed in
Germany, stored in cool caves throughout the summer and then consumed during the
octoberfest. Name it.

Ans :

8. Headquarter of which corporate was known as 'The Tower' and 'the Bunker' at
different time ?

Ans :

9. 'Fireside' is the house magazine of which company ?

Ans :
10. Which ITC product has 'sure strike' as the ad line ?

Ans :

Italy, ? , Germany, Belgium, Spain

Which name is missing in India's top five coffee exporting market ?

The answer is RUSSIA.

1. And the beast shall come forth surrounded by a roiling cloud of vengeance. The house
of the unbelievers shall be razed and they shall be scorched to the earth. Their tags shall
blink until the end of days. Where or how will you come across these lines

ANS. The Book of Mozilla, 12:10,


When about:mozilla is typed into the location bar, various versions of these browsers
display a cryptic message in white text on a maroon background in the browser window.

2. Before its 1.0 release, it had already gained acclaim from numerous media outlets,
including Forbes and the Wall Street Journal. With over 25 million downloads in the 99
days after the initial 1.0 release, it became one of the most downloaded free and open
source applications, especially among home users.What am I talking about
ANS. Firefox

3. Together with WordPerfect, this company was instrumental in making the Utah Valley
a focus for high-technology software development.. It was co-founded by George
Canova, Darin Field and Jack Davis in 1979. The name for the company Novell was
suggested by George Canova's wife who mistakenly thought that the name of the
company meant "new" in French. Which company
ANS. NOVELL.

4. It is a collection of free software tools originally developed by Cygnus Solutions to


allow various versions of Microsoft Windows to act somewhat like a UNIX system. Just
give me the name of the emulator
ANS. Cygwin

5. It is an essay by Eric S. Raymond on software engineering methods, based on his


observations of the Linux kernel development process and his experiences managing an
open source project, fetchmail. It was first presented by the author at the Linux Kongress
on May 27, 1997 and was published as part of a book of the same name in 1999. It is
commonly regarded as the manifesto of the open source movement. Name the book
ANS. The Cathedral and the Bazaar

6.The visual shows the Chief Technical Officer of a famous company.Give me the name
of the company and the CTO
ANS. Marc Ewing,REDHAT
What common honour goes to the following british companies :

Burberry, Spode, Castrol, Akzo Nobel, Corney & Barrow

The above is not an exhaustive list.

The right answer is they all have ROYAL WARRANT OF APPOINTMENT by the Queen.

Obligatory Questions:
1.Legend has it that, this man was hurt by a thorny bush as he was walking out of the city.He
bought a pot of milk(or sugar solution) and poured it on the bush so that it would be destroyed by
the ants.A ten year old boy observed this action and went on to become his student.According to
another story the man was impressed with the boy's leadership skills as he played king with his
friends in the forest and took him as a student.Who are the two people featured?
2..Green Fire shop in the Tequendama Hotel, (Bogota,Columbia) owned by Danilo Rojas raised a
huge controversy on the basis of an incident that supposedly occured on 18 May 1970.The
incident sent shockwaves through the sporting world and even Prime Minister harold Wilson
intervened to diffuse the crisis.What incident?
Scroll Down for answers.

1)Chanakya met chandragupta


2)Bobby Moore's arrest on the charge of stealing a diamond necklace from this shop,just before
the World Cup.

1)The 1st page of the 1st Tintin, "Tintin in the land of the Soviets".
(2)Bill Gates.
(3) Britney Spears.
(4) Che Guevara (at the age of 13)
(5) Allan Turing
(6) Salvador Dali.
(7) Cluedo.
(8) John Dillinger (the 1st FBI Public Enemy No. 1(9) HP computers.
(10) Charlie Chaplin.

1. X is in the Guinness book of records as the most translated author in the


world with more than 280 copies of his books in print in105 countries. X
besides being a bestseller is also a screenplay writer and has received an
academy award for "The Bachelor and the Bobby Soxer". Identify X

2. Daily occurrence ….. Where would you be using the double-slipped reef
knot?

3. Scenes for the film were shot during ___________. The actors would walk
out at the beginning or end of a match as though they were really competing.
The officials and spectators were actual officials and spectators, rather than
extras. It is the only time in the history of ___________ that this has been
allowed. The scenes showing the public queuing outside ___________ were in
fact shot outside London's zoo, simply because it looks more interesting.
Fill the dash. All three are same ;)

4. Question carried from the previous version. Put fundae…


20-6-05, 8-10-05, 28-11-05, 6-2-06.

5. This FMCG company started in early 80's was the first to introduce
shampoos in sachets there by marketing shampoos to the lower income groups
as it was not a common consumer object in the past. Headed by
C.K.Ranganathan it was known as Beauty Cosmetics then. How do we know it
now?

6. How do we know Pandit Gangadhar Vidhyadhar Mayadhar Omkarnath


shastri better?
(HINT: A famous television series)

7. Connect these cricketers:


Gul Mohammed, Amir Elahi, A. H. Kardar

8. For the HP maniacs- Fill the blanks…


______ Percival Wulfric Brain _____________

9. State the occasion for which this google daddle was put up…. (9.gif)
10. Identify this man and what was founded by him which has become
indispensable for us. (HINT: You are currently hooked on…)
(10.gif)

1.WHO WROTE THE FOLLOWING BOOKS( ALL WERE POLITICIANS)


a. The Cardinal's Mistress ********************MUSSOLINI
b The World Crisis *********************CHURCHILL
c profiles in courage ******************JFKENNEDY
2 on May 29, 1953 these words were uttered by a beekeeper
turned--------- " I and ---------(his partner) had 'knocked the
bastard off'. fill in the blank or name the event. ***** HILLARY
SPEAKING ON TENZING AND THE CONQUEST OF MT EVEREST

3 out of the 6 people in the world who were granted honarary


membership of USA only 2 were given it when alive. one was
churchill . the other a lady. who?
MOTHER THERESA?

4 during World War II, she began work in a factory spraying


airplanE parts with fire retardant. A young army photographer,
scouted local factories taking photos for a magazine article about
women contributing to the war effort. He immediately saw her
potential as a model and she was soon signed by The Blue Book
modelling agency. who are we talking about?

marylin monroe

5. we have a host of chemical elements named after countries. for a


change can you name a country named after an element?
ARGENTINA (but Abid also tells us cyprus, after copper)

6. I wrote romances under the name Mary Westmacott.but i am know as


the queen of --------

this is agatha christie but answer is QUEEN OF CRIME

. Raymonds
2. Prahalad Kakkar (he also has another cigar brand PK).
3. Mother Nature (due to all the disasters in the year 2005). It was the most popular
choice on online poll of TIME magazine for year 2005
4. The zip codes are/were exclusively for buildings. 60606 is Sears tower (Chicago),
10047-48 were for WTC
5. A company thats on at least 4 top 500/100 lists of one year (lists compiled by
FORTUNE mag - like best company, most profitable, biggest, highest growth etc)

6. Julia dreyfus (who's ex-GF of Jerry in the the sitcom) is cousin of long term serving
ceo of ADIDAS. Also some other family link. read it on wiki for details

7. Sergio Zyman (Coca-Cola, man behind flops like new Coke, OK cola etc but also big
success of the company)

Here are the answers

_____________________________________________
From: Sanjeev, Mittal (IE10)
Sent: Tuesday, February 07, 2006 3:36 PM
Subject: A biz quiz - just like that :: III

After some gap, I'm back - Sanjeev Mittal

Answers in a week

1. Which company is behind Manzoni clothing line and Million Air (a chartered air service)? ?
[HINT : "1925"]
2. "Sarso ka saag" restaurant, Shahrukh Khan's new ad for Talcom Powder and cigar Shergar -
What connects?
3. TIME person of the year 2005.. You would know. Going by the online poll they conducted, who
would have won? [HINT: Its not who but what]

4. Whats special about zip codes 60606 and 10048 etc in USA? [HINT: 10048 doesn't have what
it had till few years back]
5. Whats a Blue Ribbon Company as per Fortune magazine? It has nothing to do with Nike.
6. One from BT Acumen who missed the episode. What connect Seinfeld (sitcom) to Adidas
company?
7. Who (Famous marketing man of arguably world's most famous product company) has written
books like - The End of Marketing As We Know It, The End of Advertising As We Know It, Renovate
Before You Innovate
. The business is based in Somerset. Started in 1825,
by Quaker brothers Cyrus and James, as a business
making sheepskin rugs and slippers, using out-workers
in the village, it has expanded to become a global
shoe brand. Most famous for introduction of “Hygienic
Line”- shoes that fit the natural shape and length of
the human foot. Also the official supplier of shoes to
Britain's royal family
Name the company and its recent venture in India

2. “If there had been a capitalist down there at Kitty


Hawk, he should have shot down Orville and saved us a
lot of money." Who said this and why?

3. “The insight for this campaign is ‘disbelief’. When


someone tells us something that is too good to be
true, we often don’t believe him. Our first instinct
is to have a bet with him on it. So, we had this whole
theme of one guy believing in ___________ and the
other disbelieving him. In a way, all the commercials
so far are factual as well as entertaining, thus
coming under the genre of infotainment.” What is being
described?

4. “In our Hindi ads, we have made use of Paresh


Rawal’s voice. However, for the south, we plan to use
the Tamil film director Visu’s voice in the ad”.
Which is the ad being described here?

5. Who owns the company 'Red Chillies’? Name the super


hit movie produced ?

6. Aadhar is primarily a rural format selling fmcg


products, fertilizers, animal feed etc ideally
bringing the real rural masses into their stores while
Nature's Basket would be Urban format setup in major
capital cities. Whose initiatives are these?

7. Easy one: Identify the ad:

Brushless or lather boys,


Whichever you've a notion
Fifty cents for the ____,
A dollar for the ______.
"______ means quality,"
Said the Captain to the Boson.
So look for the package with
The ship that sails the ocean.

8. Which is the second most traded commodity in the


world, trailing only petroleum?

Delayed answers for Quiz No . 18. I regret the delay.


================
ANSWERS QUIZ # 18
================
1. CRY - Child Relief and You
2. CSIM - To Learn To Heal To Raise
3. Bill and Melinda Gates Foundation - Bringing Innovations in Health and Learning to the
Global Community
4. Amnesty International - Working to Protect Human Rights Worldwide
5. GIVE - Giving Impetus to Voluntary Effort
6. Rotary International - Service Above Self
7. Blue Cross India - Working for Animals Since 1954.
8. Together WE PREPARE - American Red Cross
9. VOICE - Helping Children Write their Future

========
[1] The group 'G' reveres Shiva as their chief god, and is involved in ancient yoga,
primitive Buddhism and Mahayanist Buddhist teachings. The group's ultimate aim is
to 'save all living things from transmigration.' The group is often referred to as a new
Buddhist sect, but it also claims to be an original religion based on Hinduism and
created by Mr. X. Identify 'X' and 'G'.
[ a Big Clue : G currently operating under the name 'Aleph,' is best known for its
March 20, 1995 terrorist attack on the Tokyo subway system.]

[2] This religious cult was was linked to an infamous 1984 criminal case in which
salmonella bacteria was used to contaminate a US restaurant's salad bars.The
devotees of this religion are sometime referred to as "Orange People". Identify the
leader of this cult.

[3] This movement was found by a Korean Minister, who claimed that Jesus had come
in his dreams when he was 15 and asked him to finish the task that was
unaccomplished after his(Jesus) crucification.Members were initially dubbed "Moon
Children" and the leader of the movement was called as "Father Moon".Critics call
the members of this movement/cult as 'Moonies'. Name the Leader and the
Movement.
[4] This symbol has been one of the best known symbols of Zoroastrian
theology.Name the symbol.

[5] This religious movement or the so called philosophy aims at "knowing how to
know" . It has attracted lots of controversies(leading it to be branded as a'cult' and
not a movement) and is at the same time been endorsed by celebrities like Tom
Cruise. (Hint : Hubbard)

1) Connect:
1) A song by the rock-band Queen
2) A famous Science-fiction novel
3) The character of Rodney Skinner in the movie adaptation of “The league of
extraordinary gentlemen”
4) A Ralph Ellison’s novel

(It’s an obscure connect!) THE INVISIBLE MAN

2) Connect: Saffir-Simpson, Fujita and Torro (A simple connect, don’t go into


complications!) ALL SCALES TO MEASURE HURRICANES AND TORNADOES

3) Which group, founded by Chung Ju-yung in 1947 as a construction company, was


once South Korea's biggest conglomerate? The company was split into five business
entities on April 1, 2003. HYUNDAI GROUP

4) Connect: Andrea, Franklin, Gaston, Ingrid, Kirk, Lee and Melissa apart from the fact
that they few of the names in the National Hurricane Center’s lists of names for
hurricanes.

THEY ARE THE NAMES WHICH REPLACED THE NAMES ALLISON,


FLOYD, GEORGES, IRIS, KEITH, LENNY AND MICHELLE IN THE LIST AS
THESE NAMES WERE RETIRED.

5) Connect Aston Martin, Bentley, Bristol, Bufori, Bugatti, Buick, Callaway, Chrysler,
Citroen, Dodge, Ferrari, Ford, Honda, Lamborghini, Lancia, Maserati, Morgan, Mosler,
Nissan, Opel, Pagani, Panoz, Peugeot, Porsche, Renault, Saleen, Skoda, Shelby, Suzuki,
Tata, Toyota among many others (apart from the clear fact that they all are names of
cars!)

ALL NAMED AFTER THEIR FOUNDER’S LAST NAMES

6) The beats for which Hollywood movie were actually made for the movie adaptation of
the novel – “A house for Mr. Mohan Biswas”, written by V.S.Naipaul? (The Bollywood
movie adaptation was never completed!) DR. NO

7) “Taking you more personally” is the tagline of which airways? QATAR AIRWAYS
8) Where in India would one find the world’s largest river island and give me its name
too? (I dedicate this one to my roommate who hails from this state) MAJULI, IN
ASSAM.

9) Which was India’s first company to feature in NASDAQ? INFOSYS

10) What is the record held by the first wife of Feodor Vasilyevi of Russia between 1707-
1782? The record is for the maximum number of children conceived. She bore 27
times – 16 pairs of twins, 7 triplets and 4 quadruplets.

) Who did you see for the first time in a series of world-wide commercials called "Been
There, Done That.�?

Ans: The Dew Dudes. The "Do the Dew" tagline appeared in 1993 - along with the Dew
Dudes - in the award-winning commercial "Been There, Done That."

2) This person was asked by a professor "If you weren't already in this business, would
you enter it today? And if not, what are you going to do about it?' What did this question
inspire him to do and who was the professor?

Ans: The Professor was Peter Drucker and he asked the question to Jack Welch of GE
who instituted the operating principle at GE that if a GE unit could not be No. 1 or No. 2
in its field, it should be jettisoned

3) She owes a lot to the Festival of India at Selfridges in London in 1982.Arguably. one
of the worst brand ambassadors for her products, she claims that she doesn't sell products
but an entire civilization in a jar. Who?

Ans :Shahnaz Hussain, Chief Executive Officer of Shahnaz Herbals-Her major turning
point was at Selfridges in London in 1982, when her products were displayed there
during the festival of India, and sold out within two hours
4) The official website declared it as smaller than a pack of gum and much more fun with
a small footnote warning consumers that it is not to be eaten! What are we talking about?

Ans: Apple's iPod shuffle

Due to its small size (8.38 � 2.49 � 0.84 cm or 3.3 � 0.98 � 0.33 inches),
"smaller than a pack of gum and much more fun,� with the footnote on its American
web site: "Do not eat iPod shuffle." As of the 29th of September 2005, the footnote has
disappeared from the American website; it remains on several international sites,
however.

5) In 2004, he "went missing," and later it was revealed that he was on a "world tour."
Upon return, he had transformed into a trimmer, slimmer character. He was the cause of a
legal scuffle between A.P. Watt and the company he served. A settlement was reached in
early 2000.In September, last year, it was disclosed by Barry Diller that he was going to
be sacked. Always a gentleman�s gentleman, he took it in his stride and is looking at
taking a few days rest before he starts his new adventure. He lives in our hearts, and
thanks to Marcos Sorenson for having made him the lovable man he was and is. Two
parts to this question.

1) What are we talking about?

2) What was the reason for the legal scuffle?

Ans: Ask Jeeves: After nearly a decade of service, Jeeves is retiring from his duties at
the search engine, which will assume the long used but little promoted name "Ask."

Askjeeves.com is now Ask.com

Jeeves was the brainchild of venture capitalist Garrett Gruener and technologist David
Warthen. From the start, Jeeves was different than the other search services of the day
and the cause of the legal battle was that, in a miss-step, the company neglected to ask
Jeeves' creator for permission to use the likeness of the character. A.P. Watt, the literary
agent responsible for P.G. Wodehouse's (the original creator) estate, had threatened
legal action against Ask Jeeves. A settlement was reached in early 2000, though neither
side disclosed details.

6) Which company originally planned to cover everything from Electricity to Energy?

Ans: Videocon � That is the reason why Videocon is shown with 2 �E�s facing
opposite directions.

7) Aspiring applicants had to demonstrate their skill in climbing the palms in the
recruitment test conducted by the board recently in Thrissur. The Cochin Devaswom
Board organized the �interview�. What job had they applied for and why this test?

Ans: Mahouts to take care of the elephants in the Cochin Devaswom .Aspiring mahouts
had to demonstrate their skill in climbing the palms in the recruitment test conducted by
the board in Thrissur. The reason: this way, they need not depend on anyone to get
coconut fronds for their pachyderms to munch on. So much for redefining multi-tasking!!

8)The typeface known as Spencerian script, developed in the mid 19th century was the
dominant form of formal handwriting in the United States during that period. In pop
culture, what arguably is the best known representation of this script?

Ans: This is the script in which Coca-Cola is written


9) It has a fantasy-berry with an embedded chip. The innumerable grains on the elegant
flesh of the berry suggest the binary language of 01010101. What are we talking about?

Ans: Bloom is the new brand identity of Wipro e-Peripherals. Designed by Shombit
Sengupta, Bloom is an aspirational concept providing a differentiating identity with
customer care, freshness and youthfulness. The Bloom has a fantasy-berry with an
embedded chip. The innumerable grains on the elegant flesh of the berry suggest the
binary language of 01010101. The chip is �tech� and reveals the rational factor in
WeP�s activity domain while the berry represents consumer�s �fantasy�.

10) Ahsaas, Bliss, Masti, Mauj, Milan, Mithun, PickMe, Sangam, Sawan, Ustad,

Zaroor, Sathi, Sparsh and Thril. Which famous brand completes this rather delectable but
incomplete list and who promotes them?

Ans: Nirodh. The above mentioned are the lesser-known brands of condoms which fall
under the Social Marketing Scheme of the Department Of Family Welfare. Social
Marketing Programme of condom was launched by the Govt. of India in 1968 with the

objective of making condoms available to those who can afford to pay nominally for
it.Under this scheme, three different varieties namely (i) New Lubricated Nirodh, (ii)
�Deluxe Nirodh� and (iii) �Super Deluxe Nirodh� are procured from the
indigenous condom manufacturers and supplied to marketing companies/NGOs (called
Social Marketing Organizations) at subsidised rates for sale in the open market.

11) Capitalised at AED 500 million, represented by 500 million AED 1 shares, all owned
by the Abu Dhabi government, the airline was established last July by a decree by His
Highness Sheikh Khalifa bin Zayed Al Nahyan, Crown Prince of Abu Dhabi and Deputy
Supreme Commander of the UAE Armed Forces.Commercial operations began on
November 12,2003 with the first scheduled flight, EY 111, from Abu Dhabi to
Beirut.What are we talking about?
Ans: Etihad Airways, designated as the �National Airline of the UAE'

12) He was born in Bristol in Gloucester, England in 1884 and so named because of his
tendency to harm visitor�s legs. He had plenty of bull terrier in him; he never hesitated
to get into a dogfight, loved chasing rats and had a fondness for the pheasants in
Richmond Park He died in the year 1895. He was buried in Kingston upon Thames, in an
area that is now the rear car park of Lloyds Bank in Clarence Street. As one enters the
bank there is a plaque on the wall stating this. It is sad to know that his road to fame
came after his death. Who?

Ans: Nipper the Dog - the HMV Logo - Francis Barraud's Dog

13)"3 pencils and quadrille pads." Who made this remarkable quip and what occasion?

Ans: Seymour Cray (1925-1996) when asked what CAD tools he used to design the
Cray I supercomputer; he also recommended using the back side of the pages so that
the grid lines were not so dominant.

14) It comes in four different versions: the "Pocket�, the 3�3�3, the "Revenge"
and the "Professor�s. It has been suggested that it's international appeal and export
achievement became one of the contributing factors in the reform and liberalization of it's
country of origin's economy between 1981 and 1985, which finally led to the move from
communism to capitalism, although some sociologists disagree. Financially, it was so
successful that its creator became the first self-made millionaire in a communist country
(according to reliable sources). What is it?

Ans: The Rubik's Cube


15) The name is slang for "moonshine." or "illicit liquor" and was originally made to be
drunk with whisky. Willy the Hillybilly was its first mascot. The first sketches of Willy
the Hillbilly that adorned these labels were created in 1948 by John Brichetto. The
product went to the extent of having old red-and-white labels featuring a hillbilly
shooting at a revenuer fleeing an outhouse with a pig sitting in the corner.The name was
intended to make the product feel like they were illegally made liquor cooked up in
regions that extend above the surrounding terrain in a limited area What are we talking
about?

Ans: Mountain Dew! Mountain Dew was born way back in the hills of Tennessee in the
1940s. Two brothers, Barney and Ally Hartman, who ran a bottling plant in Knoxville,
first trademarked the name Mountain Dew. The Hartman's Mountain Dew, however, was
a lemon-lime drink used as a mixer with whiskey. The names on the bottles are intended
to make the product feel that it was illegally made liquor cooked up in mountain stills. In
fact, the name Mountain Dew is slang for "moonshine.�

1) Connect to obtain one date/celebration that arises from the question:

http://www.flickr.com/photos/68425058@N00/144083569/

Ans: April 23- World Book and Copyright Day

April 23 was chosen as World Book and Copyright Day, with UNESCO drawing
inspiration from a Catalan tradition, where in on this day in Catalonia, numerous book
fairs and street festivals are held and customers are offered a rose with every book they
buy. It is also a symbolic date in world literature, Inca Garcilaso & Miguel de Cervantes
dies on that day while William Shakespeare was born on that day. St George’s Day is
celebrated on April 23 as well.

http://www.flickr.com/photos/68425058@N00/144087135/
2) The King of Torts went to the summons with the broker and the last juror. At a painted
house they struck a deal with the street lawyer to free the partner from the chamber.

Identify the person who emerges from this paragraph.

http://www.flickr.com/photos/68425058@N00/144083573/

Ans: John Grisham

The King of Torts, The Summons, The Broker, The Last Juror, A Painted House, The
Street Lawyer, The Partner and The Chamber are all books authored by him.

http://www.flickr.com/photos/68425058@N00/144087136/

3) When King Fahd died the flag of Saudi Arabia was not flown half-mast. Explain Why?

http://www.flickr.com/photos/68425058@N00/144083574/

Ans: Flags were not flown at half mast because the green Saudi Flag is inscribed with
Islam’s Testament of faith and lowering it would be considered improper

Some Added info: During the last FIFA World Cup the nation had objected to its flag
being printed on a ball that contained all competing nations as kicking that ball would be
unacceptable

http://www.flickr.com/photos/68425058@N00/144087137/

4) Skidmore Owings and Merrill, the American architects firm was invited to create a
very specific airport terminal in 1972. They had to factor three crucial things. High
Traffic, short period of time, diversified needs of travelers. Built with Teflon quoted
fiberglass roof units they created a solution to cover a total area of approximately 105
acres. What does this cater to?

http://www.flickr.com/photos/68425058@N00/144083575/
Ans: Haj Pilgrims who had landed at the King Abdul Aziz International Airport in
Jaddah

http://www.flickr.com/photos/68425058@N00/144087138/

5) Connect and Explain.

http://www.flickr.com/photos/68425058@N00/144083577/

Ans: Madame Tussauds

Madame Tussauds was taught the art of wax modeling by a doctor called Philippe
Curtius, she first did the model of famous author and philosopher, Francois –Marie
Arouet Voltaire. Dubai International Capital now owns The Tussauds Group.

http://www.flickr.com/photos/68425058@N00/144087139

. Which FMCG company owns Wright Pens India Ltd.?

2. Who setup the “Runway to Fame” in the Changi Airport, Singapore and what is so
special about it?

3. Identify these two people.


Pictures @ http://qfactory.ittila.info

4.Connect the following Clues

Clues available @ http://qfactory.ittila.info

5. What are these: Scotland Yard, Racing Jeep, Go to the Head of the class, WWF?

6. One of the old photos. Who are these two guys and what they are doing.
Pictures @ http://qfactory.ittila.info

7. Identify the company by the ticker symbol-HB

8. Everyone knows that it’s Shell. But why it is painted in red and yellow
Pictures @ http://qfactory.ittila.info

9. Connect Kool,Doral,Barclay and Capri ( Not an exhaustive list)


10. Connect NASCAR and P&G.

1) What national icon was created in the year 1957 to ‘express the pain of the common
people due to the spiraling prices, potholes in the roads & other day to day problems’?

Ans: The Common Man by RK Laxman

2) Which sports personality has become the first HIV/Aids ambassador for Avahan, the
Indian Aids initiative of Bill and Melinda Gates foundation?

Ans: Vishwanathan Anand

3) Which young corporate has been appointed as Chief Economist and Strategist of
Future Capital, a Unit of Pantaloons?

Ans: Roopa Purushothamam (of the famous Goldman Sach’s BRIC report)

4) Manuel des Santos Francisco was born half crippled in 1933. He went to become one
of the best dribblers of the religion called soccer. He was famous for his trademark
‘banana kick’ and ‘falling leaf’ shots. How does the world know him better?

Ans: Garrincha

5) This company started in 1847 as Telegraphenbauanstalt Von ____________ and


Halske. It grew from a small precision- engineering workshop, producing mechanical
warning bells for the railways, to one of the world’s largest companies in electrical
engineering and electronics. Some of the famous achievements of this company include
Pointer telegraph, the first electrical long distance telegraph line in Europe, the Dynamo-
Electric principle, Differential Arc lamp, Tantalum lamp and the first electric streetcar.
Also instrumental in constructing the first subway on the European continent opened in
Budapest. The founder of this company, Werner Von _________ later handed over the
company over to his brother Carl and his sons Arnold and Wilhelm, which was later
converted to a limited company. Identify.
Ans: Siemens

6) Which musician’s real name is Robert Nesta?

Ans: Bob Marley

7) This game derives from an old children’s game ‘Batttledore’. It’s named after the
country estate of the dukes of Beaufort in Gloucestershire, England, where it was first
played in about 1873. Name the game.

Ans: Badminton

8) The first issue of which magazine contained among other things, a Sherlock Holmes
story by Sir Arthur Conan Doyle, an article on the Dorsey Brothers and a feature on desk
design for the modern office called ‘Gentlemen, Be Seated.’

Ans: Playboy

9) Which organization was founded in 1961 by a British Lawyer named Peter Benenson?
Its logo is that of a candle wrapped by a barbed wire.

Ans: Amnesty International

10) It is an antivirus and computer security company headquartered in Santa Clara,


California. It markets security products and services, including the Intrushield, Entercept
and Foundstone brands. It was founded in 1987 and named for its founder John
_________. Identify.

Ans: McAfee

1) The Architecture of the Grand Hyatt Hotel in Dubai is unique in a


way, i.e the structure/layout of the building represents something. What
?
Viewed from the top, the hotel buildings resemble the word Dubai written
in Arabic
2) Irene Esther came third, Basetsana Julia came second. Who was the
first?
Aishwarya Rai, Miss world 1994

3) If it happens, who will be the youngest ever to undergo a Dope Test


(related to Athletics) ?
Buddia Singh, the 4 yr old in news currently for running marathons

4) In Dubai, a new residential city is being planned. It consists of 32


blocks painted in white and 32 painted in black. What is the name of the
city ?
Chess City

5) In Dubai (Sorry, I visited Dubai recently, that is why so many Dubai


questions!), an underwater hotel is being constructed. What is its
planned name ? (Clue: Based on a mythical underwater city)
Hydropolis

6) What is the claim to fame of the line below (along with many other
dialogues)

"Bridget is my age and lives across the street. For the first twelve
years of my life, these qualifications were all I needed in a best
friend. But that was before Bridget's braces came off and her boyfriend
Burke got on, before Hope and I met in our seventh grade Honors classes"

One of the passages from Kaavya Viswanathan's book, in news now after
being Accused of plagiarizing many sections of her book ...

Ob. Qs

This company was established during 1937 by the then ruler Nalwadi
Krishnaraja Wadiyar. It was converted into public sector undertaking
during 1947.H N Nagaraja Murthy is the current chairman and M V
Hemantha Kumar the current managing director. The company was in the
news recently. Which company is being referred here.

A: Mysore Varnishes and Paints Limited (MVPL)

What is the point on the Earth's surface at or directly below a


nuclear detonation site called in Secret Service jargon?

A : Ground Zero
Valentina Tereshkova : Sally Ride :: Yuri Gagarin : ??

A : John Glenn – first man sent by the US to outer space

P.S. These are just Ob Qs...

1. This is the Maruti (Suzuki) 800 version in Pakistan. Give me the name of the car

2. What is the name of this mascot of Indian Railways for its 150th year
celebration

3. This is the Egyptian (Islamic) version of barbie doll... name it (her)..!

4. This widely televised incident happened sometime in December. Two


Policewomen decided to teach some lessons to love-birds of Meerut. The question
is .. what is this operation nicknamed? (looking for an answer something like
operation vijay, operation yudh..)

5. I dont want the names. But who are these 4?

' I started my count at one. '

By the time we got out of the car and began walking towards the sign that said Byerly Hall: Admissions
Office, I had counted to nineteen. Reciting prime numbers always helps me relax. It was an old trick that I
used to get through important tests or presentations. It was what I did before every cello recital and
Mathletes practice.'

Which book starts with these words?

Ans:

2)

This is my dedication question of the quiz:


Identify the Turkish software engineer, whose nicknames are as follows: O-Man, Big-O, Yogi, Ivy's Bitch,
Party Animal, Kooter, O, Dostum, and Smooth.

Ans:

3)

#1- ________________

#2- Bill Clinton

#3- Alan Greenspan

Identify the person at No. 1

Ans:

4)

Since his election last year, this gentleman has been spotted wearing Serengeti-branded sunglasses and
brown walking shoes donated by Geox. He owns a specially engraved white Apple iPod, and he recently
stirred much publicity with a pair of stylish red loafers that may or may not be from Prada. Who is this
gentleman, whom marketers are seeking for their product placements?

Ans:

5)

February 2006 saw the launch of the Fashion vs Style campaign that replaced its previous 'controversial'
advertising campaign. The first advert in the campaign featured girls in the roles of Fashion and Style
fighting.

It was reported in the national press that this advert received between 121 and 127 complaints in the first
week of being broadcast and may become as controversial as the previous campaign.

What/Whose campaigns are we talking about?


Ans:

) Current Affairs:

___________, Prakash, ___________, Pratibha and Pradnya. Fill in the blanks with two
names that have stolen the front page of every newspaper recently.
Ans:

2) Etymology:

Historically this word has been used to mean blood feud. The word is Italian and
originates from Latin for 'Vengeance'. Has been recently been popularized in its own
way. Which word?
Ans:

3) How do we know 'Seeker' or the 'Student' in Persian/Arabic?


Ans:

4) Television:

"I was in Calicut and some man passing by asked me for directions. He was from
Channel [V] and was very impressed with me and told me to apply to become a VJ...
Even I thought, all this Simi, Karan, have talk shows. Why can't the common man also
host a show? So I came to Mumbai." Who is this 'beauty on duty'?
Ans:

5) Indian Music:

He made his film debut as a singer in Yahudi Ki Ladki (1933) but the songs were
scrapped and re-sung by Pahadi Sanyal. His first film as a singer was finally Sanjher
Pidim(1935). He became a music director initially in Calcutta with Rajgee(1937).
Whom are we talking about?
Ans:

6) Western Music:
How are Billy Joe Armstrong, Mike Dirnt and Tre Cool popularly known as?
Ans:

7) Business:

'Sakkat' hot Maggaa'. What are we talking about??


Ans:

Hi!
Thank You All for the excellent response to this quiz!I was initially worried that such an unconventional topic
would yield very few responses.But i was in for a pleasant surprise!
Here are the answers and scores.

1."Allah is its goal, the Prophet its model, the Quran its Constitution, Jihad its path and death for the case of
Allah its most sublime belief"
slogan of Harakat al-Muqawama al-Islamiyya aka Islamic Resistance Movement.How is it more popularly
known?
A:Hamas.Hamas is also supposed to be an Arabic word that denotes zeal or action

2.The name of this organisation literally means "bearer of the sword" in Arabic.It was reportedly founded
when Aburajik Abubakar Janjalani broke away from the Moro Liberation Front.The gang specialises in
kidnapping and extortion especially of US citizens.Identify.
A: Abu Sayyaf of Philippines.This is the group that was responsible for the kidnapping of a large
number of Americans a few years ago.

3.The organisation takes its name from a maxim of José Carlos Mariátegui, founder of Peru's first
communist party: "El Marxismo-Leninismo abrirá el ____ _______ hacia la revolución" (“Marxism-Leninism
will open the ________ _______ to revolution”)
Which organisation?

A:Shining Path founded by a University Professor named Abimael Guzman.The blank is "Sendero
Luminoso" or "Shining Path".It was active in Peru.

4. .A student named Benno Ohnesorg was killed in police firing while attending a demonstration against the
Shah of Iran.Andreas B. was the leader of a group of militant anarchists who went on a violent spree after
this incident.After he was arrested and put on trial, a journalist named Ulrike M. wrote a series of articles
sympathetic to their cause.One of her most powerful statements could be: "If one sets a car on fire, that is a
criminal offence. If one sets hundreds of cars on fire, that is political action."Both Andreas and Ulrike
supposedly committed suicide in prison (conspiracy theories say otherwise).Their organisation was called
the Red Army Faction,but its popular name was derived from the surnames of these two individuals.Which
organisation?

A:The individuals were Andreas Baader and Ulrike Meinhoff after whom the Baader-Meinhoff gang
of Germany takes its name.You would find references to them in many novels,especially those with
daring terror plots.

5.The organsiation first shot into limelight when four armed men operating in broad daylight and without the
benefit of masks, shot dead the Jordanian Prime Minister, Wash Tel, as he returned to Cairo’s Sheraton
Hotel from an Arab League meeting. The assassination itself was followed by a gruesome ritual as one of
the killers knelt down, lapped up and drank some of Tel’s flowing blood and shouted several times that he
and his accomplices belonged to ______ ______. The group then tried to assassinate Jordan’s Ambassador
to London, Zeid Al Rifai’, blew up a West German electrical installation and a Dutch gas plant. Hijacking a
Belgian Sabena airline,attacking Lod airport etc have been attributed to this organisation.But their most
stunning attack was carried out on 5 September 1972.Identify.
A:The Black September.I guess many of u figured it out from the date.The Munich massacre of
Israeli athletes occured on this date.But the group does not take its name from this date.September
1970,was the month when the Palestinian guerrillas were thrown out of Jordan by King Hussein and
his PM.The arabs dubbed this event as "Black September"

6.The organisation was founded by a student named Renato Curcio at the University of Trento.Initially they
broke into factory offices to support labourers who were opressed.In 1981,they kidnapped US Army
Brigadier General James Dozier but he was later rescued.But their most spectacular attempt was the
kidnapping and assasination of a former prime minister.Which organisation?

A:The Red Brigades of Italy also known as Brigatte Rosse.The PM who was kidnapped and
murdered was Aldo Morro.

7.Sub-Commander Marcos is currently the most influential leader of this movement.This organisation is a
much reformed version of its 1970's avatar.Infact there is even a talk of them contesting the next elections
with Marcos himself changing his name to "Delegate Zero".Which organisation/movement?

A:The Zapatistas of Mexico or Zapatista Army of National Liberation (EZLN)They are named after
Emiliano Zapata who fought for land reforms in Mexico during the Mexican revolution.More recently
they have given up violence and might join politics.

8.The Euskadi Ta Askatasuna is an organisation committed to the liberation of _______?

A:Euskadi Ta Askatasuna abbreviates as ETA.They are the people who wish to seperate Basque
from Spain.

9.In 2000, the organization changed its name to Aleph (the first letter of the Hebrew alphabet).Its earlier
name roughly translated into Supreme truth.Identify.

A:Aum Shinri Kyo,the Japanese cult founded by Shoko Asahare.They were responsible for the Sarin
gas attacks in the Tokyo subway.Aum is supposed to be derived from the Sanskrit word Om.

10.He was one of the foremost anti-communist leaders in Africa,waging war on a Soviet supported Marxist
Government.His friends called him "The Black Cockerel" . Ronald Reagen described him as a "freedom
fighter" and he was personally invited to the White House.His organisation received generous assistance
from the CIA,but its power was based more on the sale of diamonds, mined in the country and smuggled
out("conflict diamonds").Identify this leader and his organisation whose wars have ravaged an African
country.
A:Jonas Savimbi of UNITA.The country is Angola.He died a few years back.

I guess that covers pretty much every part of the globe.I skipped Al Quaeda and Laden coz it would be way
too obvious/cliched.
Hope you guys enjoyed the quiz!Will come up with something new pretty soon!!
Regards,
Manjith
"I don't feel I have to wipe everybody out, Tom. Just my enemies."-Michael Corleone

1> This Indian born American citizen, a Doctorate in Operations


Research & ex-McKinsey, supports an Indian Biotech Firm whose name
derives from the 4 base sequence of DNA (A,T,C,G). Name this Person
and the Biotech Firm ? ##

2> On a field trip with his students this Professor, a Fulbright


scholar and Doctorate in Economics, came across severe exploitation of
Bamboo artisans at the hands of Middlemen. Deeply moved by their
plight, he doled out a sum of around $30 to the artisans as an
assistance. What did this incident lead to? ##
3> Last year, this Bollywood lead actor called up the CEO of an
Entertainment Enterprise, who co-produced the movie, to enquire about
the Box Office status. The CEO replied " Pappu Pass Ho Gaya". Name
the actor and the Movie, which is being talked about. ##

4> Besides constructing an IT related SEZ, his firm is currently


involved in building infrastructure for the Translocation of an
Industry from the Indian city, which is heavily affecting its ethnic
Chinese community. Amongsts the other projects, his firm has also
built the landmark Planetarium in the city. Name the city and the Firm
##

5> During the Mid-Sixties, this young man headed the Accounts Deptt of
the US Operations of a company, founded by his Uncle. A sceptical IRS
Agent checked the company books for possible under reporting but found
the company records to be correct & the profits indeed really small.
The IRS Agent jokingly suggested closing down he US Operations and
depositing the money in a bank to get better returns!! Nearly 3 decades
later, the Accounts Head became the CEO Of the parent company. In a
span of another decade, he overhauled the production system and laid
strong emphasis on R&D. Today the Company is in the Fortune 500 list
and one of the largest recepient of patents in he US. Name this
company? ##

6> He acknowledged the Digital Generation Gap by confessing to being an


ignoramus but sounded equally confident that his daughters would be the
natives of this Age. Known to indulge in heady cocktail of Business
and Politics, he once ridiculed a religious leader for the love of
Branded Italian shoes. Name this person? ##

10 random questions, each linked to some classic piece of literature or


pop culture. Answers in a week.

1. What connects Abraham's first son, a King of Israel, a Coffee shop


chain and Led Zeppelin instrumental with a long drum solo by John
Bonham?

2. Margaret Tobin Brown was a typical rags to riches American woman who
became a major activist-philantrophists on women's issues. She was also
a survivor of Titanic and was actively helping other survivors,
especially those in the lower decks. She was even awarded a Legion of
Honour from France for this. Her comment about how she managed to
survive has led to a nickname for her which was also the title of a
Broadway Musical based on her life. What was she known as?

3. Sister of Cratus, Bia and Zelus, she is usually depicted by Ancient


Greeks as a small winged figure perching on another god's arm or the
like. She also features in similar vein in all Olympic medals. Who?

4. Banarasidas was the first Indian poet to write an autobiography. In


his work he describes the scene after the death of a monarch

<quote>
At all the houses, doors were kept closed,
merchants stopped sitting at the shops.
Nice clothes and ornaments were
all buried under the ground.

People started keeping their swords ready,


they started wearing coarse clothes.

You could not recognize the status of a person,


the rich and poor looked alike

<end quote>

Whose death and the uncertainty that was triggered following the death
is being described here?

5. Harry Houdini towards the later part of his life spent a lot of time
debunking and chasing godmen and mystics. His favourite activity was to
attend seances in disguise and debunk them. Through this activity, a
close friend of his who deeply believed in spiritualism turned hostile
and mentions his anguish and anger in the book "The Edge of the
Unknown". What is the more famous creation of this friend of Houdini?

6. Born Eva Yuonne Linda Maday-de-Maros to a Hungarian father and


Russian mother, she married an Indian army officer. She has designed a
piece of work to symbolise Rishi Dadhich's bones being used to make
Indra's Vajra. How do we know this piece of work/art?

7. Based on the Treaty of Poona signed between Hastings, the British


Governor General and Bajirao II Peshwa, a new regiment was raised at
the Peshwa's cost for the use of the British, among the rules for
raising and maintaining the regiment included rule son provision of
Nautch girls for the officers. Which highly decorated regiment is this?

8. Staying with the armies, the 93rd British Highlanders took on the
Russians at the Battle of Balaclava during the Crimean War. In order to
flank the Russians in the narrow valley, Sir Colin Campbell, the
commander, formed a two-man deep line which took on the advancing the
Russians. When a few Highlanders broke rank to fire at the Russians
before the orders, Sir Colin shouted "93rd, damn all that eagerness".
What phrase symbolising sangfroidness originates from this?

9. His name has coined a word in Hindi meaning "champion" or "extremely


skilled". In Arabic, he is known as "Dhul-Qarnayn" meaning two-horned
because in the coins minted during his time, he was depicted with horns
of a ram like the Egyptian God Amon. Who was this?

10. Finally, to round off, Connect Rang De Basanti with today i.e.
March 23rd?

) After completing his graduation, this youngster joined the family business. On a fortunate day, one of his
customers presented him with two posters that he had brought back from the US. He displayed these
posters in the shop and was amazed when visitors inquired whether the posters were for sale. Not one to let
a business opportunity pass by, he started putting up posters for sale in his shop. In 1979, he a mail order
poster shop, and started operating from a mezzanine office measuring 200 sq.ft. Identify the man and his
venture.
Ans: Anil Moolchandani and Archies
<!--[if !supportEmptyParas]--> <!--[endif]-->
2) Four brothers who left their home in Pakistan during the traumatic days of the country’s partition settled
down in Indian Punjab and started trading cycle spare parts. This company is today the world’s largest
manufacturer of bicycles. Identify the cycle company.
Ans: Hero Cycles, the flagship company of the Hero Group. The four brothers being The Munjal Brothers –
Dayanand, Brijmohan Lall, Satyanand and Om Prakash
<!--[if !supportEmptyParas]--> <!--[endif]-->
3) Living in the crowded streets of Old Delhi, Davinder Kumar Jain, understood the potential of the pen
manufacturing way back in the early 1960s. At the age of just 19, DK started producing fountain pens, a
writing instrument that deeply fascinated him, thus laying the foundation of the largest manufacturer of
writing instruments in India. Identify the company/brand.
Ans: Luxor, which produces over 1,000,000 pens a day
<!--[if !supportEmptyParas]--> <!--[endif]-->
4) This magazine’s ranking of India’s most valuable companies is considered the Indian equivalent of the
Fortune 500 listing. Identify the brand
Ans: Business Today
<!--[if !supportEmptyParas]--> <!--[endif]-->
5) This international company’s Indian business was set up initially as a small operation in Konnagar (near
Calcutta) in 1931. In January 1934, it acquired a plot of 155 acres from the Calcutta Port Trust and from
adjacent landowners. The foundation stone for the first building of their operation – now called the _______
Clinic – was laid on October 28th 1934. In the years that followed, the overall site was doubled in area.
Identify the brand.
Ans: Bata. The township is today popularly known as Batanagar
<!--[if !supportEmptyParas]--> <!--[endif]-->
6) More than two decades ago, Dr. Prathap C. Reddy, a successful medical practitioner in the United States
lost a patient who could not make it to Texas for an open-heart surgery. That was the day when Dr. Reddy
mooted the idea of setting up a healthcare facility that would offer world-class service to millions of people in
India and the subcontinent. Thus setting up one of India’s best healthcare centers.
Ans: Apollo Hospitals- Touching Lives’
<!--[if !supportEmptyParas]--> <!--[endif]-->
7) Connect Turner Morrison, Tata Coffee Company, C.Sivasankaran and Bean2Cup to an Indian
superbrand.
Ans: Barista
<!--[if !supportEmptyParas]--> <!--[endif]-->
8) Which over the counter antiseptic cream carries the line ‘Kin to your skin’
Ans: Boroline

1. Eric Ambler’s 1962 novel The Light of The Day was


made into this popular 1964 heist movie starring Peter
Ustinov and Maxmillian Schell. The plot deals with a
small time crook Abdel Simpson getting involved with a
pair of jewel thieves in Turkey. Name the movie.

2. This 1997 movie starring Clint Eastwood and Gene


Hackmann is based on a novel by David Baldacci. It
deals with the story of a small time crook Luther
Whitney who is an eye witness to a crime committed by
US President, and the subsequent cover up operations.
Name the movie.

3. This novel was written by the author when he was in


bed suffering from illness. It was his first thriller
and introduced the character of Richard Hannay. The
dealing with conspiracy theory set in WW1 was a great
hit with the soldiers then.

4. Born in Baltimore, he was son of a police


detective. He was commissioned as a Second Lieutnant
in US Marine Corps. After being seriously injured in a
helicopter accident, he left the Navy and worked for
Merill Lynch as a stockbroker. He married Caroline
Muller, a medico student and did his Phd in History
from Georgetown University and later worked as a
history teacher at US Naval Academy. He is credited
with inventing the Canary trap, a method for exposing
an information leak. Name this character.

5. Justin Quayle finds his wife Tessa murdered in


Nairobi. Investigation reveals a conspiracy involving
pharma companies and a corporate scandal. Also made
into a movie starring Ralph fiennes, name the book.

6. “Pride and Extreme Prejudice, “The Price of the


Bride”,”A Casualty of War” and “A Little Bit of
Sunshine” are the 4 novellas in this 1991 book. These
4 stories are actually narrated by a former SIS agent
Sam Mc Ready during his trial. Name the book.

7. Typical John Grisham novel about a young law


graduate Rudy Baylor, he joins as an associate to
Lyman “Bruiser” Stone a successful ambulance chasing
lawyer. In his quest for picking up accident victims,
he meets Deck Shifflet an unethical ex insurance
surveyor.

8. A “Da Vinci Code” sort of novel by Robert Ludlum,


it centers around the Order of Xenope, a remote Greek
Monastic brotherhood, which has some ancient
manuscripts related to the death of Jesus Christ. And
also a tale involving two brothers fighting over the
document.

9. The Chancellor Manuscript by Robert Ludlum, is the


story of an author, who uncovers a plot by Invar to
assassinate a real life figure who was quite
controversial during his time. Which personality?

10.
"But -" I paused. "Good God, Gregori, no sane man, not
even the most monstrous criminal in history, would
ever dream of such, of such - In the name of heaven,
man, you can't mean it!"
"It may be that I am not sane," he said.
The above quote is taken from a very popular Alistair
McLean novel. Name the book which IMHO, is one of
McLean’s best.

).the addresses 495 Grove St. Apt. 19, New York, NY 10001,495 Grove
St. Apt 20, New York, NY 10001,5 Morton St. Apt. 14, New York, NY
10001 appear in which popular sitcom...
the orignal name of the sitcom was "The Six of Us" after which it was
changed to the present form
the sitcom friends

2).what are easter eggs in gaming technology.....


An Easter Egg is a hidden feature that
the programmers have put in their software.They should
be
hiden,unobvious,entertaining,non-malicious,reproducible,and
put there by the producers for their personal reasons
reasons,eg the Pinball Game in MSWord 97.
3).what are chalkboard gag and couch gag....
in the simpsons the starting visuals have bart writing on a
blackboard(he writes a diffrent thing everyday) and the whole family
sitting on a couch ...
4).this company was founded as a manufacturer of test andmeasurement
instruments with a US$500 investment in a Palo Alto, CA garage by two
graduates of stanford university .Their innovation was the use of a
small night-light bulb as a temperature dependent resistor in a
critical portion of the circuit which they sold for $54.40....which
famous company am i talking about...
hewllett packard.. it is also the first successfull company of the
silicon valley
5). Born in Manchester, Jamaica, he emigrated from Jamaica to Canada
at age 13, and played basketball before his graduation from Queen
Elizabeth Park School in Oakville, Ontario. He began competing as a
100m sprinter part-time in 1991, but he didn't take up the sport
seriously until 1994. At that time, he was also a successful
stockbroker. who....
donovan bailey
6).connect count olaf , bruce nolan,hank evans, Fletcher Reede and
Lloyd Christmas...
characters played by jim carrey:- count olaf is a series of unfortunate
events,
bruce nolan is bruce almighty,hank evans is me,myself and irene,
fletcher reede is lair!lair!and lyold christmas is dumb and dumber
7).which language can be written left to right , right to left, top to
bottom , bottom to top and
diagonally in any direction...think fiction
liliputians :-the language of liliputs:-moon gogoi was the only person
to crack it ...congrats

8).after whom has been the diplomatic enclave in delhi named after...
chanakya and the enclave is chanakyapuri
9).he was a brother of Alpha Phi Omega, a service fraternity and Kappa
Kappa Psi, a band service fraternity.he also made made history by
being the first sitting President to speak to a gay rights
organization.... who?
bill clinton

10).connect Graeme Hick, Andrew Caddick, Geraint Jones,Andrew Strauss


and Kevin Pietersen ....

all players not born in england

.Easy one to start with:


Whose body remains were found in 1997 after 30 years of his death at Valegrande during the
construction of an airport?
Che Guera.
2.Which country gives away the "Order of the Elephant"?
Denmark.
3.How do you better know Charles Edouard Jeanneret-Gris?
Le Corbusier.
4.Whose first film appearance was in the 1942 film "Pahili Mangalagaur"?
Lata Mangeshkar.
5.Identify this personality (See attachment).
Simon Wiesenthal.(Only Rajasekhar got it right.)
6.Which sport is called "Famfrpal" in Czech,"Zwerkbal" in Dutch,"Huispas" in Finnish,"Rumpeldunk" in
Norwegian,"Metlobal" in Slovak and "Quadribol" in Portugese?
Quidditch.
7.He lost the Nobel prize for literature in 1957 to Albert Camus only for one vote.He started writing only at
the age of 51, yet he was able to produce some great novels.Apart from his mother tounge, he writes in
Creton and French.
Who am I talking about?
Nikos Kazantzakis.
8.Which scientist was present inside the Enola Gay on its journey to Hiroshima?
Nobody got this. The answer is Luis Walter Alvarez.
This nobel laureate invented the detonator for the atombomb, Ground control approach
system for aeroplanes, the bubble chamber to detect radiation, presented the theory that the
dinosaurs became extinct after the impact of an asteroid on earth and was a scientific advisor to the
Earl Warren commssion.
9.Which great explorer set off to his historic journey on a ship called "Fram"?
This question has two answers- Roald Amundsen and Fridtjof Nansen.
The ship was actually Nansen's ,but he allowed Amundsen to use his ship to travel to the South
Pole.
10.Who ws the first sportsperson to be featured on a currency note?
Paavo Nuurmi.

There are approximately 30 million of them all over India. Most of them
are
named after the place where they are located or after the person who
established them there. They fall into three categories. The first are
self-generated. They are believed to be the most sacred and are said to
have
fallen from heaven. Next are fixed . These are crafted and then, after
an
initiation ceremony done according to Puranic rites, installed at a
sacred
location. The third category is Jangam, or movable. Which is the first
and
what are we talking about?

Lingas and Svayambhu Linga; Lingaraja Temple, Bhubaneshwar. This is


exemplified by the fact, that the presiding deity, here, is the
Svayambhu
Linga - half Shiva, half Vishnu, a unique feature of the temple.

What did N G Bhansai and Hancock design?


Ans: The Filmfare Award Statuette

This serial was supposed to be called Amma, but that was changed at the
dress designer's request to its current name. The protagonist was once a
Miss India contestant, and now reportedly earns $405000 a year, the
highest
for an Indian actor. Which never-ending soap is this?

a.Kyunki Saas Bhi Kabhi Bahu Thi

'Modern Times 'was claimed to be a remake of a French Movie 'A nous la


liberte' by Rene Clair. Since it was made under a German banner J
Goebbels
wanted to sue Chaplin but couldn't as Rene Clair refused to support him.
Why?

Rene Clair refused saying that he was honoured to have inspired


Chaplin!!

Born with the body of a mastiff, looks of a lamb, teeth of a bunny


rabbit
and is one of those mutations that happen when God plays dice. Who or
What
are we talking about?

Ronaldo

US banned her entry calling her 'a communist good'. She was kept in On
her
journey to the USA from China she got into trouble when the London for
time's sake and became a major attraction there. Who?
Chi-Chi the Panda, who became the mascot of the WWF

Many years ago a certain insect called the Mexican Beetle was imported
into
Bangalore in large numbers. Why?

To eradicate the weed Parthenium

When he ran the NY Marathon to represent Sikhs, people were still


calling
him "Bin Laden" while he was running. He came to London in 1992 to live
with
his son after his wife's death in his village in Jalandhar. 'Sitting at
home
was really killing,' he says in Punjabi. 'Most elderly people in
Britain eat
a rich diet, don't move about and only travel in cars, and that makes
them
sick,' he says. He wasn't prepared to go the same way. So he took up
jogging
initially to beat the boredom of sitting at home. Who is he?

A: Fauja Singh. He replaced David Beckham as Adidas's new poster boy.


On
the latest Adidas billboards, spread across London, he is sitting
cross-legged; as if cooling his heals after a day's work out, peeping
over
Londoners, tired and overworked, telling them the secrets of his
unfailing
energy. Fauja Singh, Britain's most popular Sikh is 93...

The oldest of the species is supposedly more than 600 years old and is
found
in Kerala's Perambikulam game sanctuary. Now it is a primary component
for
reforestation and its bark and flowers are used as medicine for
Bronchitis.
Its name originated in Malayalam and entered Portuguese to mean
'carpenter'.
Which tree?

The Teak

Carl Sagan headed a research team called TTAPS which introduced a


phrase
into the English language. What phrase?

Nuclear Winter

What contribution did a movie by name 'Rooplekha' make to Indian


cinema?

First movie with a flashback sequence

OLYMPIA FINALS (IIT DELHI) Answers given below.

1. ___A__ began to study acting during his infamous 9 month ban in


1995. His filmography is:

Le bonheur est dans le pr� - 1995 - Lionel

Eleven Men Against Eleven - 1995 - Player (uncredited)

___B____ - 1998 - Monsieur le Foix

Mookie - 1998 - Antoine Capella

Les enfants du marais - 1999 - Jo Sardi

La grande vie! - 2001 - Joueur de p�tanque 2

L'Outremangeur - 2003 - S�l�na


La vie est � nous - 2005

Une belle histoire - 2005

A and B?

2._______ (October 6, 1888 – October 5, 1918) was an early French


aviator and a fighter aircraft pilot during World War I.

________ was already a noted aviator before the war. In 1913 he gained
fame for making the first nonstop flight across the Mediterranean Sea.
The next year he joined the French army at the outbreak of World War
I. After several missions he decided that shooting and flying at the
same time was too difficult, so he fitted a machine gun to the front of
his plane so the tasks became one and the same. In order to protect
the propeller from the bullets, he fitted metal wedges to the prop.
Starting from April 1, 1915, he soon shot down three German planes
and quickly garnered an excellent reputation.

_______ managed to escape from prisoner-of-war camp in Germany in


February 1918 and joined the French army again. On October 5th,
1918, he was shot down again and killed near Vouziers. _____ was an
outstanding aviator and probably the first real fighter pilot in the world.

3. The DLF Golf and Country Club, India's first night Golf Course is
today ranked as the best in the Country as also amongst the leading
Golf Courses in Asia. It is the first golf course with night golfing
facilities. This course has been designed by a very famous golfer and
very successful business executive who was named "Athlete of the
Decade" for the 1960s in a national Associated Press poll. He had won
92 championships in professional competition of national or
international stature by the-end of 1993. Sixty-one of the victories
came on the U.S. PGA Tour, starting with the 1955 Canadian Open.
Who?
4. About defeating his Idol Rudy Hartono in the Swedish open he says
“I could have beaten him on ‘love’ in the last game, but I did not have
the heart to do that to my Idol. So I ensured that he got one point
before I beat him,” A teetotaller to the core, when asked to celebrate
another famous victory by drinking champagne, he diluted it with
apple juice and then drank it. Who is this great sportsperson?

Hint: "Touchplay", his biography

5.� Born in West Baden Springs, Indiana, the son of Georgia and Joe
____. He grew up in both West Baden and the adjacent town French
Lick, which earned him the nickname "the Hick from French Lick" in his
later career. He led the Indiana State University (ISU) Sycamores to the
NCAA championship game in 1979, his senior season, only to lose to
the Michigan State University Spartans, who were led by his future NBA
rival, Magic Johnson. He was a member of the all star “Dream Team"
which won the men's basketball gold medal at the 1992 Olympics. He
also starred in the movie "Space jam".

Michael Jordan, who may have supplanted him as the league's most
feared clutch player through his heroics with the Chicago Bulls, once
was asked who he would want to take a shot with the game on the
line, other than himself. Before the question could be finished, Jordan
quickly responded, "_______"

6.� Born in Brantford, Ontario, Canada on 26 Jan 1961. On his 18th


birthday, he was signed in by Peter Pocklington, owner of the
Edmonton Oilers for a 21 year old contract. During his NHL career
spanning two decades, he had played for 4 different teams, namely
Edmonton Oilers, Los Angeles Kings, St. Louis Blues, and New York
Rangers. He has appeared on the cover of the Time magazine along
with Larry Bird. He is currently a part owner of the Phoenix Coyotes.
Name him.

7.� Connect the three Pics.

Hint to this Pic -

___________ is the name given to the shooting of seven people as part


of a Prohibition Era conflict between two powerful criminal gangs in
Chicago, Illinois in the winter of 1929: the South Side Italian gang led
by Al Capone and the North Side Irish/German gang led by George
'Bugs' Moran.

8. Initially, he competed mostly in the 180-yard hurdles and 440-yard


dash. Before March, 1976, he ran only one (400-meter hurdles) race,
but once he began focusing on the event he made remarkable
progress, so much so, that he qualified for the US team for the 1976
Summer Olympics in Montreal.

Though it was his first international meet, _____ won the gold medal
and set a world record of 47.64 seconds. He was the only American
male in Montreal to win an individual track and field gold medal.
From Sept, 1977 to June, 1987 he had won a mind-blowing 122
consecutive races. Who?

9. Born May 28, 1938 in Chelyan, West Virginia, he was called "Zeke
from Cabin Creek". He attended East Bank, West Virginia High School
from 1952-1956. He was named an All-State from 1953-56, and an All-
American in 1956, when he was also named West Virginia Player of the
Year after becoming the state's first high-school player to score more
than 900 points in a season (32.2 ppg, 1956). On November 26, 2005,
his number 44 became the first basketball number to be retired by
West Virginia University.

This apart, how has he been immortalized in NBA?

Hint –

10. Fauja Singh is a 93 year-old who took up running to cope with his
wife’s death. He was signed for a certain company’s famous campaign
along with David Beckham and Laila Ali. He is the oldest competitor in
the London Marathon and holds the record in the 90 plus age bracket.
At the age of 89, he completed it in 6 hr54 min. At 92, he did it in 5 hr
40 min. Which company and campaign?
11. X became a GM at the age of 15 years and 4 months, beating the
previous record for youngest Grandmaster, set by Bobby Fischer in
1958. X's father believed that "Geniuses are made, not born" which is
why X was made to go through special training at home. On the April
2003 FIDE ratings list, X's 2715 rating made X the number 10 ranked
player in the world. That same year, X scored X's greatest victory: an
undefeated clear 2nd place in the Category 19 Corus chess
tournament in Wijk aan Zee, Netherlands, just a half-point behind Y,
and a full point ahead of world champion Z. X?

12. At the 1972 10000m Olympic final� this athlete bumped into an
American athlete and fell to the ground dragging a Tunisian athlete
Mahomad Gammoudi along with him, while the Tunisian lay there
stunned this athlete recovered to win the gold in world record time,
bringing his country's first track gold in 36 yrs. He also won the 5000m.
He repeated the feat at the 1976 Olympics. Who is this great athlete?

13. The Brookhaven Bulletin published a story on employee William


Higinbotham on March 13, 1981 speculating that he may have
invented the first video game in 1958. Creative Computing magazine
picked up on the idea and published it in an October 1982 article,
crediting Higinbotham as the inventor, that is, until they heard from
someone who could document an earlier game. The same story was
reprinted in the Spring 1983 issue of Video and Arcade Games, a sister
magazine to Creative Computing. To date, no one has been able to
prove an earlier claim. Which game did he invent and what was the
medium of projection?

14. This game was invented in 1895 by William G. Morgan, a student at


Springfield College and a director of the YMCA at Holyoke,
Massachusetts who wanted to create a game for his classes of
businessmen which would demand less physical contact than
basketball. It was originally called "Mintonette". It was introduced as an
Olympic sport in 1964 Tokyo Olympics. Which game?
15. Connect:

Giacomo Agostini (Italy) - motorcycling

Severiano Ballesteros (Spain) - golf

Franz Beckenbauer (Germany) - football

Boris Becker (Germany) - tennis

Ian Botham (England) - cricket

Sergey Bubka (Soviet Union/Ukraine) - athletics

Bobby Charlton (England) - football

Sebastian Coe (England) - athletics

Nadia Comaneci (Romania) - gymnastics

Deng Yaping (China) - table tennis

David Douillet (France) - judo

Emerson Fittipaldi (Brazil) - motor racing

Sean Fitzpatrick (New Zealand) - rugby union

Dawn Fraser (Australia) - swimming

Tanni Grey-Thompson (British) - paralympic athletics

Tony Hawk (USA) - skateboarding

Miguel Indur�in (Spain) - cycling

Michael Johnson (USA) - athletics

Michael Jordan (USA) - basketball

Kapil Dev (India) - cricket


Kip Keino (Kenya) - athletics

Franz Klammer (Austria) - skiing

Dan Marino (USA) - American football

John McEnroe (USA) - tennis

Edwin Moses (USA) - athletics

Nawal El Moutawakel (Morocco) - athletics

Robby Naish (USA) - windsurfing & kiteboarding

Ilie Nastase (Romania) - tennis

Martina Navratilova (USA) - tennis

Jack Nicklaus (USA) - golf

Pel� (Brazil) - football

Gary Player (South Africa) - golf

Morn� du Plessis (South Africa) - rugby union

Hugo Porta (Argentina) - rugby union

Viv Richards (West Indies) - cricket

Mark Spitz (USA) - swimming

Daley Thompson (England) - athletics

Alberto Tomba (Italy) - skiing

Katarina Witt (Germany) - figure skating

Yasuhiro Yamashita (Japan) - judo

16. Born Artur Antunes Coimbra in 1953, was the youngest of five
footballing brothers. He made his league debut for Flamengo in 1973
and his international debut in 1976, against Uruguay, scoring with one
dead-ball shots for which he became famous. He netted over 100 goals
in his first two seasons and was South American Player Of The Year in
1977 (and in 1981 and 1982).

In the 1978 World Cup he suffered niggling injuries and was unhappy
with coach Claudio Coutinho's defensive game. In 1982 however, ____
returned to the attacking style which suited his change of pace, body-
swerves and dynamic shooting. His hat-trick against Bolivia clinched a
place in the 1982 finals and in Spain his four goals, including the
equalizer against Scotland, took his total for _____ past 50. In 1983
after 650 goals and four _____ championship medals, he made a
�2.500.000 move to Udinese, returning to Flamengo in the summer
of 1985.

He played in three games in the 1986 World Cup, all as a substitute.


His last match was the quarter-final against France when ______ fans
chanted for him. Alas, minutes later he missed a penalty. It was a sad
end for one of ______'s most popular players of all time. After 1047
senior games, including 71 for ______, he eventually retired in 1990 and
was appointed his country's Sports Minister.

Answers –

1. Eric Cantona, Elizabeth

2. Roland Garros

3. Arnold Palmer

4. Prakash Padukone, after he defeated king in the 1981 all England


champs final, this incident occurred

5. Larry Bird

6. Wayne Gretzky

7. Jake la Motta

8. Edwin Moses
9. Jerry West ; (His dribbling silhouette has long been used in the
National Basketball Association's official logo.)

10. Adidas "Impossible is Nothing"

11. Judith Polgar, Y – Anand, Z - Kramnik

12. Lasse Viren

13. Tennis for two and oscilloscope

14. Volleyball

15. Laureus World Sports Academy

16. Zico

1. The Battle of Mactan was fought on April 27,1751 by


Lapu Lapu a local tribal chieftain of the island of
Mactan against Spanish colonizers. The battle was
chronicled by Antonio Pigafetta. Who was killed in
this battle?
FERDINAND MAGELLAN

2. Set in Puritan New England in 17th Century, this


novel narrates the story of Hester Prynne, who gives
birth due to an adulterous affair and the consequent
fall out. It gets it’s title from the practice of
having an “A” on her clothing to identify her as an
adultress during those times.
THE SCARLET LETTER

3. Born in Worcester, he was highly impressed by


H.G.Well’s The War of the Worlds, and this fuelled his
interest in rocketry. In 1926 he launched the first
liquid fuelled rocket at Auburn in Aunt Elfie’s farm.
Using a music rack as launcher he developed the basic
idea of the bazooka. His theories on the probability
of inter space travel using rockets was ridiculed by
the media. Ironically, while the US Army rejected his
designs, it was the Nazis through Werner Von Braun
who developed the V2 rockets based on his designs and
he is often called as one of the “fathers of modern
rocketry”.
ROBERT GODDARD

4. One of the volcano islands, north of Guam, it gets


it’s name from a local world meaning “Sulphur Island”
due to the huge sulphur deposits located on it. It
was the site of a major battle during WW2, and ended
with the Americans capturing Mt. Suribachi, the
island’s highest point. Name the island.
IWO JIMA

5. On March 17,1970 14 officers of the US Army were


suspended for their role in this event during the
Vietnam War. In 1971 Lt. William Calley was convicted
for ordering this massacre, and the photos of this
incident in fact were a major catalyst in turning
public opinion against the war. Name the incident.
MY LAI

6. This place gets it’s name from a Greek Word meaning


“Holy Echo” and is a very sacred city for the Jews.
Believed to be the oldest inhabited settlement in the
world. It was the first city to be handed over to
Palestinian Authority in 1994 after the Gaza
agreement. After Israel reoccupied it, it was again
handed over to Palestine in 2005. Name the place.
JERICHO

7. On March 16,2005 Ripudaman Singh Malik and Ajaib


Singh Bagri, were found not guilty on all counts by a
Canadian court. What crime were they accused of?
BOMBING OF AIR INDIA FLIGHT KANISHKA

8. Born in Erlangen, Germany, he became Maths


professor at Jesuits College, Cologne in 1817. And
later in 1852, professor of experimental physics at
University of Munich. His first major publication was
his 1827 pamphlet titled Die galvanische Kette
mathematisch bearbeitet and is one of the major works
dealing with electricity.
GEORGE OHM

9. Nicknamed as Beppo, this Nazi officer was called as


Todesengel or Angel of Death by concentration camp
inmates. One of his famous quotes was “the more we do
to you, the less you believe we are doing it”. Famous
for his experiments on prisoners, he escaped to Brazil
after the war, and was drowned at a beach in Embu in
1979 while swimming.
JOSEF MENGELE

10.
This song is set to the melody of the Bulgarian “Sadi
Moma” and a metal version was written by Jono Bacon.
It belongs to the genre of filk and begins with the
lines
“Join us now and share the software;
You'll be free, hackers, you'll be free.
THE FREE SOFTWARE SONG
1.D w a y n e van der Slui s an d Andr e w Aker s are cre d i t e d to be th e inv e n t o r s
of thi s sp or t . It inv ol v e s a per s o n g e t t i n g stra p p e d up insi d e an en or m o u s
pla s t i c air cu s h i o n e d ball an d goi n g do w n for a quick roll do w n a slo p e . The
dia m e t e r of th e sp h e r e is aro u n d 10 to 12 fe e t de p e n d i n g on th e nu m b e r of
pe o p l e harn e s s e d insi d e . What do you call thi s sp or t origi n a t e d in Ne w
Zeal a n d ?

Ans .

2. Much accl ai m e d Italia n dire c t o r Nan ni Morat ti' s film ,"Th e Cay m a n" is
ba s e d on th e life of whi c h st a t e s m a n ?

Ans .

3."Its bi g g e r th a n I tho u g h t it wa s ," he s ai d of th e 14 an d half inch ,1 8 cara t


trop h y . Who sai d thi s d e s c ri b i n g wh a t ?

Ans .

4.H o w is lea d gui t ari s t Saul Hud s o n of th e rock ban d Velv e t Rev ol v e r
po p u l arl y kno w n as ?

Ans .

5.B o t h Tig er Woo d s an d Arjun Atw al ar e m e m b e r s of whi c h golf club ?

Ans .

6.Litt e r a t e u r Man o h a r Shy a m Joshi wh o pa s s e d aw a y rec e n t l y tran s f o r m e d


India n Tel e v i s i o n en t e r t a i n m e n t with whic h 2 s eri al s ?
Ans .

7. Iden ti f y th e ani m a l in th e pict ur e . It is a noc t u r n a l ani m a l whi c h fe e d s on


an t s an d ter mi t e s .

Ans .

8. Sel m a n Abrah a m Wak s m a n dis c o v e r e d str e p t o m y c i n in 19 4 4 , to tre a t


tub e r c u l o s i s . He wa s aw ar d e d th e Nob e l Priz e in phy s i o l o g y in 19 5 2 . Wha t
co m m o n l y us e d ter m did he firs t us e in de s c ri b i n g str e p t o m y c i n ?

Ans .

9. Iden ti f y thi s per s o n , wh o fou n d e d a m e d i c a l re s e a r c h ins ti t u t e na m e d


aft e r him in th e 19 5 3 .

His act u a l prof e s s i o n or lin e of bu si n e s s ha s not be e n m e n t i o n e d s o as not


to mak e th e an s w e r to o ob vi o u s

Ans .

10. ____ _____ is origi n a ll y a sp or t s w r i t e r' s ter m , but it ha s not hi n g to do


wit h th e nu m b e r four. The ter m wa s firs t us e d in 19 3 7 by th e Ne w York
Her al d - Tribu n e sp or t s w r i t e r Cas w e l l Ada m s . He us e d it in ref e r e n c e to th e
un o f fi ci al con f e r e n c e of te a m s als o kno w n as th e Old Ten . Fill up th e blan k s

Ans .
11. Dev e l o p e d by Wilbur Sc o vi ll e in 19 1 2 wh a t do e s th e Sc o v ill e Heal t h Unit
or SHU m e a s u r e ?

Ans .

12. The lea d perf or m e r of th e rock ban d , "The Ordin ar y Fear of God",
cre a t e d quit e a con tr o v e r s y by s m o k i n g on st a g e durin g a live perf or m a n c e
in his nati v e cou n t r y flou ti n g anti tob a c c o law s . Who is th e p er s o n in
qu e s t i o n ?

Ans .

13. Which Sou t h Am eri c a n ho s t s th e lat e ni g h t cha t sh o w , "La Noc h e Del


Die z"?

Ans .

14. Saif Ali Khan no w hol d s th e dis ti n c t i o n of be c o m i n g th e firs t India n to


ap p e a r on an Aus tr ali a n po s t a g e st a m p . His perf or m a n c e at th e con cl u d i n g
cer e m o n y at th e 20 0 6 Com m o n w e a l t h Gam e s in Melb o u r n e wa s fe a t u r e d on
th e st a m p .

Give m e th e na m e s of eit h e r of th e tw o film per s o n a l i t i e s bef o r e him wh o


w er e fe a t u r e d in a po s t a g e st a m p of a for ei g n cou n t r y

Ans .

15. Which s m all pacifi c isla n d' s na m e m e a n s - "look , a coc o n u t" . It ha s a


hig h l y profit a b l e do m a i n na m e (lik e .in m e a n s India) on th e int er n e t whi c h
m e a n s – "now" in Sw e d i s h . But thi s do m a i n na m e wa s sol d to a form e r
co m p u t e r ma g a z i n e edi t o r (A m e r i c a n ) , thu s de n y i n g th e go v e r n m e n t of
us e f u l rev e n u e

Ans .
16. W h a t w er e th e s e 8 e m i n e n t per s o n s ref e rr e d to as . The log o in th e
back g r o u n d ma y s er v e as a clu e

Ans

17.Th e fam o u s log o of whi c h corp or a t e wa s de s i g n e d by Prof e s s o r Vau g h a n


Pratt of Sta n f o r d Univ e r s i t y

Ans .

18. W h a t con n e c t s th e 3 pe o p l e sh o w n in th e vis u a l bel o w ap ar t fro m all


bei n g writ e r s or po e t s of accl ai m .

Ans

19.Id e n t i f y th e sp or t s w o m a n in th e pict ur e and her clai m to fa m e

Ans .

20. W h i c h co m p a n y ' s log o is ref err e d to as th e "coff e e ring"?

Ans

. Name the first person from Asia to fly into space.

Ans : Tuan Pham from Vietnam

2. Which king of England was known as 'the sailor king' ?

Ans : William IV
3. Which MCC team toured West Indies without winning a single match ?

Ans : The 1947-48 team led by G O Allen ( 2 draw and 9 loss )

4. Which Indian alcoholic drink reminds you of 'gypsy' ?

Ans : Zingaro

5. Which celebrity actor has two pet dogs named 'binka' and 'nugget' ?

Ans : Daniel Radcliff

6. What is the phrase ( chosen after by a biblical hero ) for 'a person who intends to
comfort but in the process increases distress'.

Ans : A Job's Comforter

7. In Mahabharat, whose son was Laxman ?

Ans : Duryodhan

8. 't-l-l-i-' .... fill in the blancs to form a word meaning'pain in heel or ankle'.

Ans : Talalgia

9. Which airline was chosen the best international career for the year 2005 ?

Ans : Cathay Pacific ( chosen by 'skytrax' )

10. 'Dressing the world' tagline of which apparel brand ?

Ans : Digjam

1. “SUN rises here “ .. wat s the importance of this phrase? The phrase has
something to do with the picture!. Identify the connection.(
http://image005.mylivepage.com/chunk5/75802/64/1.1.jpg)

The place in the picture is Niagara falls and SUN(Stanford University Network)’s next
revolutionary product is the Ultra sparc microprocessor named NIAGARA. Eric Schmidt
of google quoted that SUN rises in the market with this new product.
Hence the image , SUN rising at the Niagara falls.

None cracked it! Pretty tough I suppose…


2. This image indicates a very famous company!!!. The company’s name has got
the credit of most useful word in the computer era. Identify the company and the
relation with the picture!
http://image005.mylivepage.com/chunk5/75802/64/1.2.jpg)
The company is google … the backtracking of web pages is the concept and google was
initially named BACKRUB…

Almost all got this right !!!!

3. whose tagline is this? “Let your Heart Fly”.


Indian(Indian airlines)

4. This personality owns a famous engineering college in India. And stands 11 th in


a famous list from India. Identify the personality and the college he owns?
The personality is Shiv Nadar (HCL) he owns SSN (siva subramaniya nadir college of
engg.) in tamil nadu,India. As per forbes list of richest people in India, shivnadar stands
11 th…..

only balaji rao cracked this que(good work !!!!!!)

5. Identify the importance of this posture…


http://image005.mylivepage.com/chunk5/75802/64/1.5.jpg
This is the front cover of the times magazine … they are bill&mellinda gates and singer
Bono of U2 being chosen as Man of the year by time magazine …

6. how is “marshall mathers ” known now?


A sitter , every body got it . its EMINEM ,
Marshall Mathers were called M&M which further colloquially became eminem

7. which of the following was not the name of a worm, virus or bug to affect users of
Microsoft's products during the past few years?
(a)CodeRed (b) ILOVEYOU (c) GatesSmasher (d) Melissa(e) MyDoom.
Answer is GatesSmasher.. I know our quizzing minds will think as the name
GATESSMASHER is related to Microsoft… that is why I posted this que(he he )

8. When there are two full moons in the same month, what is the second called?
Blue moon (again a sitter)

9. Celine Dion - "A new day has come"-- whats the controversy???
This album was the first album to be released in a copyrighted cd that cannot be played
on a computer to avoid piracy…. Once again none cracked it . but a nice que is nt it?

10. What is the unit of mouse movement?


mickey
11. identify the personality! With the first picture . If you cant, you take the clue in
the second picture!

http://image005.mylivepage.com/chunk5/75802/64/1_11_1.JPG

http://image005.mylivepage.com/chunk5/75802/64/1_11_2.jpg

the person in the image is GEORGE LAZENBY , a short lived James bond .. (only one
film !)

the image 2 is a novel “on her majesty’s secret service ”, the name of the movie he
acted!!!
12. Identify the owner of the voice! And the famous movie which screened the audio?
http://file005.mylivepage.com/chunk5/75802/57/Sound%201.wma

the owner of the voice is ELTON JOHN, for the movie lion king …
a nice tune, with good lyric and pictured like two lions duet! Nice to

1)Who did playback for the poems 'Udal Mannuku' and 'Unoodu njan' in
the Maniratnam movie 'Iruvar'?

Aravind Swami

2)Which is the only film in which the 'big 3' of malayalam movies
Mamooty, Mohan Lal and Suresh Gopi appeared.
(i think this is the only one)

Manu Uncle(basically a childrens movie in which Mamooy was the main


star, Mohan Lal has a guest role and Suresh Gopi who was not a top
hero at the time appeared as a cop)

3)Which actress has acted opposite Chiranjeevi, Nagarjuna, Venkatesh,


Kamal Hassan, Rajni Kanth , Mohan Lal , Vishnuwardhan and Amitabh
Bachan

Soundarya

4)Vinodh Khanna and son Akshaye has both acted opposite this actress
.who?

There are two ...Dimple and Madhuri

5)Who played Agent M in Men InBlack 2?

Michale Jackson
(both Ramasubramanian G. S and Varun Reddy said that he just
appears on screen pleading that he'd like to be part of the MiB, and
that he could be agent M... i framed this from IMDB page for the
movie where he has been casted as agent m..sorry for that slip)

6)Shot in B&W , this was a travalogue on Rajasthan produced by the


govt and went on to win India's first golden bear in berlin film fest.
Id the movie and director

Through The Eyes of A Painter by MF Hussain

7)He started his film career with the silent movie "Raiders of the
rail road" in 1936.He is more remembered for a television seriies made
in late 80's that started a range of similar productions. Who?

Ramanand Sagar

8)He is the only person to have acted in the first talkies made in
Hindi, Tamil and Telugu. Who?

L V Prasad

9)What was special about the role played by Maggie Smith in the movie
'California Suite' that won her the best supporting actress oscar in
the year 1978?

Played the role of an actress losing oscar(has awarded half points for
mentioning oscars)

10)About whom was the movie 'Darshan the Embrace' that was showcased
in cannes 2005 based on?

Mata Amrithanandamayi

1) This art-form or style of fighting, more appropriately


stick-fighting , is practiced in Southern India -
especially in Tamilnadu(and also in Kerala).Hand and foot
movements along with good reflexes are mandatory for this
form of fighting.In this art, when the Master recognizes
the Student has mastered the curriculum, only then he gives
the red scarf to acknowledge him as a Master. Hence no
"black-belt syndromes"! This form of fighting can also be
seen in many Kollywood movies(esp during the 70-80s era).

Answer : Silambattam (Silambam)

(2) 'X' was instrumental in the downfall of the Mughal


Empire.He was well known for his forts and guerrilla
tactics that he used successfully to conquer and annexe
many regions and provinces.He is also remembered for being
the only secular king in medieval India. An organization
has also adopted a new religion and made him the principal
deity.Identify 'X'.[ Hint : Mumbai's International Airport
was renamed in his honour]

Answer : Chhatrapati Shivaji Raje Bhonslé

(3) Her debut feature film won her the Golden Camera award
at the Cannes Film Festival and was also nominated for an
Oscar.She is a New York-based Indian film director.Her
latest project is Maisha, a film lab to help East Africans
and South Asians learn to make films. Her last master work
Monsoon Wedding, a film about a chaotic Punjabi Indian
wedding, was awarded the prestigious Golden Lion award at
the Venice film festival.Identify this female.

Answer : Mira Nair

(4) 'Z' happens to be a distinguished Atomic and Nuclear


scientist .He studied cosmic rays under the Nobel laureate
Dr.C. V. Raman at the Indian Institute of Science (IISc),
Bangalore.The Indian space Program was his brainchild - he
can be clearly called as the Father of the Indian space
Program. He was also involved in the creation on IIM
,Ahmedabad. Identify this National Hero and a Great Indian
Physicist.

Answer : Vikram Sarabhai

(5) 'X' was born in Indore, Madhya Pradesh, into a family


with strong ties to the film industry. His father Salim
Khan is a successful screenplay writer. One of Salim Khan's
wives (he has two) is the famous dancer Helen. X's brothers
P and Q are also actors, if not as successful as X. X has
has two younger sisters, one married to actor Atul
Agnihotri.X has also been in the news for some
controversial reasons.[Trivia : He always appears in films
wearing a bracelet with a turquoise stone; his father wears
a similar "lucky" bracelet.]

Answer : 'X' = Abdul Rashid Salim Salman Khan popularly


known as Salman Khan; 'P'= Sohail Khan ; 'Q' =Arbaaz Khan

1.WHO WROTE THE FOLLOWING BOOKS( ALL WERE POLITICIANS)


a. The Cardinal's Mistress
b The World Crisis
c profiles in courage

2 on May 29, 1953 these words were uttered by a beekeeper turned------


---
" I and ---------(his partner) had 'knocked the bastard off'. fill in
the blank or name the event.

3 out of the 6 people in the world who were granted honarary


membership of USA only 2 were given it when alive. one was
churchill . the other a lady. who?

4 during World War II, she began work in a factory spraying airplane
parts with fire retardant. A young army photographer, scouted local
factories taking photos for a magazine article about women
contributing to the war effort. He immediately saw her potential as a
model and she was soon signed by The Blue Book modelling agency. who
are we talking about?

5. we have a host of chemical elements named after countries. for a


change can you name a country named after an element?
6. I wrote romances under the name Mary Westmacott.but i am know as
the queen of --------

) Scientists have applied emotion recognition software that measures a person's


mood by examining features such as the curve of the lips and the wrinkles
around the eyes.

The findings show that XXX was 83 per cent happy, 9 per cent disgusted, 6 per
cent fearful, and 2 per cent angry, according to the British weekly New Scientist.

Identify XXX.

Ans: Mona Lisa. The computer software, developed by Nicu Sebe at the
University of Amsterdam and researchers at the University of Illinois, examines
key facial features, the journal reports.

<!--[if !vml]--><!--[endif]-->

2) Incorporated on Jan 10, 2000 as M/s Orbis Infotech Private Limited at New
Delhi under the Companies act, 1956 with registration number 55-103183, its
name was changed to the present name on March 16,2001. Identify the
company.

Ans: IndiaBulls. Founded by Sameer Gehlaut and Saurabh Mittal

<!--[if !supportEmptyParas]--> <!--[endif]-->

3) Which Indian company (well known in a completely different business) is now


heading into the real estate business under the name `Rose Garden Housing
Developers Private Limited'?

Ans: BPL (British Physical Laboratories). Their first apartment block is coming
up on Bannerghata Road, Bangalore. This is after selling their dry cell battery
unit to Eveready and signing a Joint Venture with Sanyo for their color
televisions. BPL is also all ready to bring out low-end mobile phones by March
2006. R&D on these phones is done at Nambiar International MicroTech, a
wholly owned subsidiary of BPL.

<!--[if !supportEmptyParas]--> <!--[endif]-->

4) Bombay: Summer of 1967. A Charni Road flat. Mrs. Sheela Mane, a 28-year-
old housewife is out in the balcony drying clothes. From her second floor flat she
can see her neighbours on the road. There are other people too. The crowd
seems to be growing larger by the minute. Unable to curb her curiosity Sheela
Mane hurries down to see what all the commotion is about. She expects the
worst but can see no signs of an accident. What had drawn the interests of the
crowd?

Ans: It was the first Amul hoarder..

<!--[if !supportEmptyParas]--> <!--[endif]-->

5) Around 150 years ago, Dr. Campbell, a civil surgeon, planted tea seeds in his
garden at Beechwood, 7000 ft above sea level as an experiment. He was
reasonably successful in raising the plant because the government, in 1847,
elected to put out tea nurseries in this area. This was the beginning of a very
famous brand. Which brand?

Ans: Darjeeling Tea. According to records, the first commercial tea gardens
planted out by the British tea interests were Tukvar, Steinthal and Aloobari tea
estates. This was in 1852 and all these plantations used seeds that were raised
in the government nurseries

<!--[if !supportEmptyParas]--> <!--[endif]-->

6) Climacool, Barricade and Supernova are brands from which stable?

Ans: Adidas. Other products are Traxion, Adiprene, Torsion and Adidas 1.0

<!--[if !supportEmptyParas]--> <!--[endif]-->

7) The term `Leap Ahead' is to replace a phrase which had become trademark of
the company using it.. What phrase is `Leap Ahead' going to replace?

Ans: Intel Inside

<!--[if !supportEmptyParas]--> <!--[endif]-->

8) Identify:

http://qkphotos.blogspot.com/2006/01/bizzbliss18-question8-identify.html

Ans: Gianni Versace

Answers
1. Raymonds
2. Prahalad Kakkar (he also has another cigar brand PK).
3. Mother Nature (due to all the disasters in the year 2005). It was the most popular
choice on online poll of TIME magazine for year 2005
4. The zip codes are/were exclusively for buildings. 60606 is Sears tower (Chicago),
10047-48 were for WTC
5. A company thats on at least 4 top 500/100 lists of one year (lists compiled by
FORTUNE mag - like best company, most profitable, biggest, highest growth etc)

6. Julia dreyfus (who's ex-GF of Jerry in the the sitcom) is cousin of long term serving
ceo of ADIDAS. Also some other family link. read it on wiki for details

7. Sergio Zyman (Coca-Cola, man behind flops like new Coke, OK cola etc but also big
success of the company)

1.Eric Ambler’s 1962 novel The Light of The Day was


made into this popular 1964 heist movie starring Peter
Ustinov and Maxmillian Schell. The plot deals with a
small time crook Abdel Simpson getting involved with a
pair of jewel thieves in Turkey. Name the movie.
TOPKAPI

2. This 1997 movie starring Clint Eastwood and Gene


Hackmann is based on a novel by David Baldacci. It
deals with the story of a small time crook Luther
Whitney who is an eye witness to a crime committed by
US President, and the subsequent cover up operations.
Name the movie.
ABSOLUTE POWER

3. This novel was written by the author when he was in


bed suffering from illness. It was his first thriller
and introduced the character of Richard Hannay. The
dealing with conspiracy theory set in WW1 was a great
hit with the soldiers then.
THE 39 STEPS

4. Born in Baltimore, he was son of a police


detective. He was commissioned as a Second Lieutnant
in US Marine Corps. After being seriously injured in a
helicopter accident, he left the Navy and worked for
Merill Lynch as a stockbroker. He married Caroline
Muller, a medico student and did his Phd in History
from Georgetown University and later worked as a
history teacher at US Naval Academy. He is credited
with inventing the Canary trap, a method for exposing
an information leak. Name this character.
JACK RYAN

5. Justin Quayle finds his wife Tessa murdered in


Nairobi. Investigation reveals a conspiracy involving
pharma companies and a corporate scandal. Also made
into a movie starring Ralph fiennes, name the book.
THE CONSTANT GARDENER

6. “Pride and Extreme Prejudice, “The Price of the


Bride”,”A Casualty of War” and “A Little Bit of
Sunshine” are the 4 novellas in this 1991 book. These
4 stories are actually narrated by a former SIS agent
Sam Mc Ready during his trial. Name the book.
THE DECEIVER

7. Typical John Grisham novel about a young law


graduate Rudy Baylor, he joins as an associate to
Lyman “Bruiser” Stone a successful ambulance chasing
lawyer. In his quest for picking up accident victims,
he meets Deck Shifflet an unethical ex insurance
surveyor.
RAINMAKER

8. A “Da Vinci Code” sort of novel by Robert Ludlum,


it centers around the Order of Xenope, a remote Greek
Monastic brotherhood, which has some ancient
manuscripts related to the death of Jesus Christ. And
also a tale involving two brothers fighting over the
document.
THE GEMINI CONTENDERS

9. The Chancellor Manuscript by Robert Ludlum, is the


story of an author, who uncovers a plot by Invar to
assassinate a real life figure who was quite
controversial during his time. Which personality?
EDGAR HOOVER

10.
"But -" I paused. "Good God, Gregori, no sane man, not
even the most monstrous criminal in history, would
ever dream of such, of such - In the name of heaven,
man, you can't mean it!"
"It may be that I am not sane," he said.
The above quote is taken from a very popular Alistair
McLean novel. Name the book which IMHO, is one of
McLean’s best.
THE SATAN BUG

1) ) Which movie made by Deepa Mehta was inspired by


the book “The Ice Candy Man” by Bapsi Sidhwa, a female
Pakistani writer residing in the USA? Her next plan is to
novelize another movie made by Deepa Mehta. 1947
EARTH (REMEMBER THE ICE CANDY
CHARACTER IN THE MOVIE?)

2) Who became the first singer to exceed 1 million digital


downloads? She was given a party in Japan in honour of
her achievement. GWEN STEFANI (HER SONG
“HOLLABACK GIRL” BECAME THE FIRST SONG
TO BE LEGALLY DOWNLOADED A MILLION
TIMES)

3) Which organization was formally set up by Keshavrao


Baliram Hedgewar on 27th September, 1925 at Nagpur,
which has remained the organization's headquarters since
then? THE RASHTRIYA SWAYANSEVAK SANGH
(FONDLY CALLED AS RSS, SOME WERE CLOSE
WITH VHP)

4) Risk Addiction is the sequel to which famous movie?


BASIC INSTINCT (THIS WAS SUPPOSED TO BE A
SITTER! VERY FEW GOT IT RIGHT THOUGH!)
5) What was called Stevie’s little wonder by TIME when it
was released by APPLE’s CEO Steve Jobs. I-POD NANO
(I HAVE GIVEN A DOUBLE FOR THE FULL
ANSWER, SUPPOSED TO BE THE TOUGH ONE,
THOUGH MANY ANSWERED IT CORRECTLY
AND FULLY!)

6) “Riding car”-made in 1885, a one-cylinder wonder made


by Benz, is considered to be the world’s first what?
MOTORCYCLE (VERY FEW GOT THIS RIGHT)

7) Name the person associated with this quote: “How my


mother longed for a day to see me as a grown up man
by putting me in my father’s shoes. As I grew, I became
barefoot.” M.F.HUSSAIN (SOME GUESSED
GANDHI! IT WAS A REALLY GOOD GUESS!)

8) Who will become the President of the English football


association next year? PRINCE WILLIAM (VERY FEW
ANSWERS FOR THIS ONE!)

9) Brainchild of Zimbabwe-born Scottish writer Alexander


McCall Smith, Mma Precious Ramotswe is which
country’s only and finest female fictional detective.
BOTSWANA (IT WAS SUPPOSED TO BE THE
TRICKY ONE. THOUGH MANY GOT IT RIGHT!
CHEERS TO ALL THOSE GUYS! I ALSO GOT
SOUTH AFRICA AS ANSWERS!)

10) Mindcomet is a Florida based company that is beaming


what into space so that aliens will read them and better
understand ordinary humans? BLOGS (WHAT WILL
THEY “READ”? THAT IS THE CLUE!)
Brand Misplacement question::-)

Qn Number 0: In the Tamizh movie, Tamizhan starring "Ilaya Thalapathy"


Vijay
and Priyanka Chopra. There is a fashion show where "Comedy King" Vivek
tries
to escape from the cops and hence participates in a fashion show and is
solicited by the Hotel Manager. In return he asks the Manager for a few
minutes. How many minutes and why? Alternatively, give me Vivek's name
is
his avatar as a female and what's the funda behind the name?

Answer: Two minutes. Vivek in his female avatar calls himself "Maggi"
from
Bombay and the cajoles the inspector to give him two minutes because
like
Maggi, he will be ready for the Manager, in 2 minutes

1) When Lincoln was assassinated, one of the Agency's employees drew a


eulogy to the president in the margin of the page that contained the
report
on Lincoln's law office. A cross-like tombstone with the vertical
letters 'A
L' and a weeping willow tree were inked in black, in a small box, the
inscription: "This office had the honor of having Old Abe as a
correspondent. Which Company? (Incidentally, He was one of the four U.S.
Presidents who served as a business correspondent for this company,
often
laced his reports with humor. On one of them, he described a grocer in
his
home town of Springfield, Illinois as possessing a rat hole in his shop
that
"would bear looking into.")

Dun & Bradstreet

2)Herzogenaurach is a small town in Mittelfranken, Bavaria.This city (of


approximately 24,000 inhabitants) is over 1000 years old. It is located
about a 30 minute drive from Nuremberg. It was first mentioned in a
document
in the year 1002 under the name of Uraha. So why does it figure in this
business quiz?

It is the birthplace of the sporting good giants Adidas and Puma

3)Boble (bubble) in Norway,Fusca in Brazil; Coccinelle in France;


Maggiolino
in Italy; Sedán or Vocho in Mexico; Kever in the Netherlands; kotseng
(literally, 'hunchback')/"pagong" (turtle) in the Philippines; Garbus
(literally, 'Hunchback') in Poland; Brouk in Czech Republic; Carocha in
Portugal. What was it called in the U.S. and what are we talking about?

Volkswagen Beetle called 'The Bug' in the U.S.

4) Their most popular models were represented in the 3,500-strong


official
vehicle fleet including Centennial and XG sedans as well as Terracan and
Santa Fe SUVs for VIP use. The total sponsorship package was valued at
Euros
16.3 million, including automobile supply, services and cash for the
exclusive rights. What are we talking about?

When the 28th Olympic Summer Games opened in Athens Greece, the
international media spotlight once again shone on Hyundai which served
as
the official vehicle supplier. Under the terms of the Grand National
sponsorship agreement, Hyundai Hellas in partnership with Hyundai Motor
Co.
supplied the Athens 2004 Organizing Committee for the Olympic Games
(ATHOC)
with a fleet of Hyundai sedans and minivans to meet the immense
transportation requirements of the tens of thousands of athletes, VIPs,
officials and journalists.

6)Both men were successful in their careers, working hard all their
lives.
Yet one struggled financially all his life. The other would become one
of
the richest men in Hawaii. One died leaving tens of millions of dollars
to
his family, charities and his church. The other left bills to be paid.
Who
is talking about what?

Rich Dad, Poor Dad is a book by Robert Kiyosaki that discusses what
Kiyosaki
feels rich parents teach their children that poor or middle class
parents do
not.

Kiyosaki writes:

"I had two fathers, a rich one (infact, he was his frienf Mike's dad)
and a
poor one. One was highly educated and intelligent; he had a Ph.D and
completed four years of undergraduate work in less than two years. He
then
went on to Stanford University, the University of Chicago, and
Northwestern
University to do his advance studies, all on full financial
scholarships.
The other father never finished eighth grade"

7)In traditional usage, one has a small amount of foam is spooned onto
the
espresso; at Starbucks/Coffee Day and those coffeehouses which follow
their
lead, the order is reversed for some beverages, with espresso added to a
large volume of foam on top of steamed milk. The other is an espresso
based
drink with a volume of milk, from 6-12oz (180-360mL) total volume, with
less
foam than a cappuccino. Name these stimulants and the person who is an
expert at brewing them.

Macchiato, Latte and the brewer is called a Barista

8) A city in Washington is named after him. He used his money and


success to
develop a comfortable railroad sleeping car. The first one was finished
in
1864. Although the sleeper cost more than five times the price of a
regular
railway car, by arranging to have the body of President Abraham Lincoln
carried from Washington, D.C. to Springfield on a sleeper, he received
national attention and the orders began to pour in. By 1865, he wanted
to
expand business into the reconstructing South and sent Kimball to act
as his
Southern agent. Pullman built a new plant on the shores of Lake Calumet,
several miles from Chicago. In an effort to make it easier for his
employees, he also built a town with its own shopping areas, theaters,
parks, hotel and library for his employees. Name him and which mega-
entity
made him famous in quizzing circles?

George Mortimer Pullman. UPS is well-known for its brown trucks,


internally
known as package cars (hence the company nickname "Big Brown") The brown
color that UPS uses on its vehicles and uniforms is called Pullman
brown,
after the railroad sleeper cars created by George Pullman, which used
the
same color.

9) Y&R opened an office in Chicago in 1931, and recruited him sometime


later
to create an advertising research department. Young was forced out in
1934.
Rubicam retired in 1944. Identify this legend that started the
department.
He remained with Y&R for sixteen years.

George Gallup of the Gallup polls fame. His doctoral dissertation was
entitled A New Technique for Objective Methods for Measuring Reader
Interest
in Newspapers. In 1958 Gallup grouped all of his polling operations
under
what became The Gallup Organization.

10) 1: The society after agrarian revolution and replaced the first
hunter-gatherer cultures.
2: Nuclear family, factory-type education system and the
corporation
3: ???
What is 3 or explain?

Alvin Toffler's Third Wave concept.


The post-industrial society. Third Wave is the post-industrial society.
Toffler would also add that since late 1950s most countries are moving
away
from a Second Wave Society into what he would call a Third Wave
Society. He
coined lots of words to describe it and mentions names invented by other
people, like the Information Age.

Scroll down for information about: RV QuizCorp's Under The Peepal Tree -
2006
www.rvquizcorp.com
-----------------------------------------------------------------------
-----
-----------------------------------------------------------------------
-----
--------------------------------------------------------------------

From: Avinash Mudaliar [mailto:avinashmudaliar@3xus.com]

Time to switch channels...

Tune into this year's episode of RV QuizCorp's Under The Peepal Tree -
2006.

The challenge to compete against India's best quizzing brains is not


something that can be talked about; you have to experience it for
yourself.
So, be there at India's largest and unique Quiz Fest - UTPT-2006.

We would like to dedicate this year's Under The Peepal Tree (UTPT '06)
to
Wing Cmdr Mulky (Retd,), a beacon and the guiding force behind RV
QuizCorp.
Without his encouragement neither would we have been quizzing or
started RV
QuizCorp.
He was the light who showed us all what quizzing is, what it means and
more
importantly what the essence of knowledge is. He left behind memories
of his
own humility, a spark of his essence, a promise to man of what he could
become, with effort...In short he left us hope...
He will be one person who all of us can only aspire to be, but will
never
be...
This is our miniscule tribute to the pioneer who defined and redefined
many
a knowledge seeker's life.
May, his soul rest in peace.

Sir, your memory remains...

P.S.
Apart from the winners of each quiz taking home a booty, there are
loads of
informals to keep your brains ticking.

DAY 1: 24th March, Friday

The Major Quiz (A General Quiz) - 9 A.M.

Indi-Genius ( The India Quiz ) - 1 P.M

DAY 2: 25th March, Saturday

Sci-Tech Quiz- 9 A.M

Sports Quiz - 1 P.M.

DAY 3: 26th March, Sunday

Audi Vidi Vici ( Audio-Video Quiz, Open for All)- 9 A.M.

Fine Answers-The Corporate General Quiz ( only for "Corporates") - 2 P.M

VENUES

For the College Quiz: RV Teacher's College Auditorium, Jayanagar.


College Quizzes: Teams of THREE, and registrati

Answers here:

1. How much do you need to pay for one Loonie in Canada?


Ans: 1 canadian dollar (its nickname is loonie)

2. Connect Kothari Products, Coco Cola and Roopalben Panchal


Ans:YES (WATER brand, coca cola Ad line, Yes Bank IPO fraud)
3. Whats the name of girl on Amul hoardings?
Ans:Dimples - (not sure on this.. can anyone confirm?)

4. Bunty Aur Bablee you have heard of. Which brand uses Babla and Bunny?
Ans:Lijjat papad

Hello All,

Before I give you the solution, let us go through the puzzle again.

PROBLEM :

In an American University english language class for foreign


students, which stipulates a minimum of 7 numbers of students from any
community for eligibility, there are students only of Indian,
Pakistani, Korean, Chinese and Japanese nationality. Indian, Pakistani
and Koreans never bunked their classes. Here are some facts about the
five groups..

1. Chinese are the only group with odd number in count.


2. Koreans say that all except 93 in their class were Koreans
3. Chinese say they never saw any Japanese in their class. Japanese
say likewise.
4. Indians were equal to Koreans and Pakistani students combined in
strength
5. Japanese say all except 84 in their class were Japanese.
6. Only one community has less than 10 students. Koreans are the
second in minority with four more than the least.

How many students are enrolled in the class ? What is the profile ?

SOLUTION :

Let us assume number of students of .....

Indians : I
Pakistanis : P
Koreans : K
Chinese : C
Japanese : J

Chinese and Japanese were never present in the class together.


Therefore when Koreans say 'all except 93 in their class were Koreans,
that's the strength of I, P and C or J.
I + P+ C or J = 93 ( statement 2 ) .... i

similarly

I + P + K = 84 ( statement 5 ) .... ii

Subtracting ii from i, we can derive that C or J is 9 more in number


than K

Stetement i says Chinese are the only group with odd number. So

C = K + 9 .... iv

Again I = P + K ( statement 4 ) .... iii

substituting iii in ii, we get

2 x I = 84 or

I = 42

There is only one group with less than 10 students ( statement 6 ).


It will have to be an even number ( statement 1 and Chinese are the
only odd numbered community ). It will have to be 8 as minimum number
of any community has to be 7. It can be either Pakistani or Japanese. So

P or J = 8

Koreans are second minority with 4 more than the east. So

K = 12 ( as it is, strength of Koreans could be minimum 11 ( 7+4 )


and maximum 13 ( 9+4 ) as the least group has strength between 7 and 9
( single digit ). Chinese being the only group having odd number
strength, only strength Koreans can have is 12 ).

from iv, we get

C = 21

from ii, substituting values of I and K, we get

P = 30

so J = 8
therefore the students profile in the class is:

I = 42
P = 30
C = 21
K = 12
J = 8

............
113

Total number of students is 113

NAEF IMAM was the only person to crack it. It is really great on his
part to be unique.

Cheers!

Sudip Roy

This hill station nested in the Shivaliks is widely


believed to have taken it’s name from a Tibetan word
meaning “Assembly of King’s ministers”. After the
Anglo Bhutan war came to an end with the Treaty of
Sinchula in 1840, this place was ceded to the British
who developed it as a hill station. Name the place
KALIMPONG-MANY GAVE THE ANSWER AS DARJEELING, WHICH
MEANS "LAND OF THUNDERBOLT"

2. Dharamsala in Himachal Pradesh is located on this


mountain range, which gets it’s name from the meaning
“White Mountain”. These ranges are inhabited by the
Gaddi tribe.
DHAULADHAR-NO POINTS FOR DHAULAGIRI

3. This Indian hill station contains a large number of


Playwin lottery centers, and in fact the Govt of the
state earns a lot of revenue through authorized
gambling. Which place.
GANGTOK- AGAIN NO POINTS FOR SIKKIM, I HAD ASKED THE
NAME OF THE TOWN, NOT THE STATE

4. Legend has it that Lord Hanuman rested here on his


way to fetching the Sanjeevani herb. It also contains
the Pasteur Institute established in 1900 by Sir David
Semple is one of the oldest and produces anti rabies
vaccines. Which place?
KASAULI-LARGE NUMBER OF GUESSES INCLUDED SIMLA, OOTY

5. In 1819 two civil servants of the Madras Govt


Whish and Kindersley, on a trip to Nilgiris, came back
and reported the “discovery of a table land with
European climate”.. It was developed by John Sullivan,
and take’s it’s name from the tribes living in the
hills around it.
KOTAGIRI-AGAIN A LOT OF OOTY ANSWERS, KOTAGIRI WAS
NAMED AFTER THE KOTA TRIBES IN THAT AREA

6. This hill station is called the “Chiki” capital of


India, as its famous for the large number of outlets
manufacturing this sweet there. Which one?
LONAVALA

7. Discovered by Hugh Malet in May 1850, it was


developed by Lord Elphinstone. It derives it’s name
from a local word meaning “Mother Forest”. The place
has the famous Lake Charlotte and also it’s monkeys.
MATHERAN- SOME GAVE THE ANSWER AS YERCAUD, WHICH IS
DERIVED FROM TAMIL WORD MEANING "LAKE FOREST"

8. This lake located in Mt.Abu is believed to have


been dug by the nails of the Gods, and gets it’s name
from that legend. Name it.
LAKE NAKKI(POINTS GIVEN FOR NAKKI JEEL ALSO)

9. In 1827 Capt Young a British military officer


explored and developed this place. The place is
believed to have got it’s name from the native shrub
that grew in that vicinity.
MUSSORIE- SOME WRONGLY GAVE THE ANSWER AS KODAIKANAL-
KODAI IS FAMOUS FOR KURINJI FLOWER, BUT THAT HAS
NOTHING TO DO WITH PLACE NAME

10. Often called as the “Scotland of India” due to


it’s similarity with the Scottish highlands. The town
gets it’s name from the name of the local deity,
meaning Superpower of God, who is believed to be
dwelling on the mountain peak that overlooks this
place.
SHILLONG- AGAIN NO POINTS FOR MEGHALAYA. I ASKED THE
NAME OF THE TOWN SPECIFICALLY

Hussain-2
Anagha-2
Shafeek-3
Charan-3
Amlan-5
Paresh-4
Abir-9
Gaurav-9
Ravi Bhatia-3
Shrikant-6
Samrat-6
Nikhil-5
Bala-7

__________________________________________________________
Yahoo! India Matrimony: Find yoThis hill station nested in the
Shivaliks is widely
believed to have taken it’s name from a Tibetan word
meaning “Assembly of King’s ministers”. After the
Anglo Bhutan war came to an end with the Treaty of
Sinchula in 1840, this place was ceded to the British
who developed it as a hill station. Name the place
KALIMPONG-MANY GAVE THE ANSWER AS DARJEELING, WHICH
MEANS "LAND OF THUNDERBOLT"

2. Dharamsala in Himachal Pradesh is located on this


mountain range, which gets it’s name from the meaning
“White Mountain”. These ranges are inhabited by the
Gaddi tribe.
DHAULADHAR-NO POINTS FOR DHAULAGIRI

3. This Indian hill station contains a large number of


Playwin lottery centers, and in fact the Govt of the
state earns a lot of revenue through authorized
gambling. Which place.
GANGTOK- AGAIN NO POINTS FOR SIKKIM, I HAD ASKED THE
NAME OF THE TOWN, NOT THE STATE

4. Legend has it that Lord Hanuman rested here on his


way to fetching the Sanjeevani herb. It also contains
the Pasteur Institute established in 1900 by Sir David
Semple is one of the oldest and produces anti rabies
vaccines. Which place?
KASAULI-LARGE NUMBER OF GUESSES INCLUDED SIMLA, OOTY

5. In 1819 two civil servants of the Madras Govt


Whish and Kindersley, on a trip to Nilgiris, came back
and reported the “discovery of a table land with
European climate”.. It was developed by John Sullivan,
and take’s it’s name from the tribes living in the
hills around it.
KOTAGIRI-AGAIN A LOT OF OOTY ANSWERS, KOTAGIRI WAS
NAMED AFTER THE KOTA TRIBES IN THAT AREA

6. This hill station is called the “Chiki” capital of


India, as its famous for the large number of outlets
manufacturing this sweet there. Which one?
LONAVALA

7. Discovered by Hugh Malet in May 1850, it was


developed by Lord Elphinstone. It derives it’s name
from a local word meaning “Mother Forest”. The place
has the famous Lake Charlotte and also it’s monkeys.
MATHERAN- SOME GAVE THE ANSWER AS YERCAUD, WHICH IS
DERIVED FROM TAMIL WORD MEANING "LAKE FOREST"

8. This lake located in Mt.Abu is believed to have


been dug by the nails of the Gods, and gets it’s name
from that legend. Name it.
LAKE NAKKI(POINTS GIVEN FOR NAKKI JEEL ALSO)

9. In 1827 Capt Young a British military officer


explored and developed this place. The place is
believed to have got it’s name from the native shrub
that grew in that vicinity.
MUSSORIE- SOME WRONGLY GAVE THE ANSWER AS KODAIKANAL-
KODAI IS FAMOUS FOR KURINJI FLOWER, BUT THAT HAS
NOTHING TO DO WITH PLACE NAME

10. Often called as the “Scotland of India” due to


it’s similarity with the Scottish highlands. The town
gets it’s name from the name of the local deity,
meaning Superpower of God, who is believed to be
dwelling on the mountain peak that overlooks this
place.
SHILLONG- AGAIN NO POINTS FOR MEGHALAYA. I ASKED THE
NAME OF THE TOWN SPECIFICALLY

Hussain-2
Anagha-2
Shafeek-3
Charan-3
Amlan-5
Paresh-4
Abir-9
Gaurav-9
Ravi Bhatia-3
Shrikant-6
Samrat-6
Nikhil-5
Bala-7

__________________________________________________________
Yahoo! India Matrimony: Find yoThis hill station nested in the
Shivaliks is widely
believed to have taken it’s name from a Tibetan word
meaning “Assembly of King’s ministers”. After the
Anglo Bhutan war came to an end with the Treaty of
Sinchula in 1840, this place was ceded to the British
who developed it as a hill station. Name the place
KALIMPONG-MANY GAVE THE ANSWER AS DARJEELING, WHICH
MEANS "LAND OF THUNDERBOLT"

2. Dharamsala in Himachal Pradesh is located on this


mountain range, which gets it’s name from the meaning
“White Mountain”. These ranges are inhabited by the
Gaddi tribe.
DHAULADHAR-NO POINTS FOR DHAULAGIRI

3. This Indian hill station contains a large number of


Playwin lottery centers, and in fact the Govt of the
state earns a lot of revenue through authorized
gambling. Which place.
GANGTOK- AGAIN NO POINTS FOR SIKKIM, I HAD ASKED THE
NAME OF THE TOWN, NOT THE STATE

4. Legend has it that Lord Hanuman rested here on his


way to fetching the Sanjeevani herb. It also contains
the Pasteur Institute established in 1900 by Sir David
Semple is one of the oldest and produces anti rabies
vaccines. Which place?
KASAULI-LARGE NUMBER OF GUESSES INCLUDED SIMLA, OOTY

5. In 1819 two civil servants of the Madras Govt


Whish and Kindersley, on a trip to Nilgiris, came back
and reported the “discovery of a table land with
European climate”.. It was developed by John Sullivan,
and take’s it’s name from the tribes living in the
hills around it.
KOTAGIRI-AGAIN A LOT OF OOTY ANSWERS, KOTAGIRI WAS
NAMED AFTER THE KOTA TRIBES IN THAT AREA

6. This hill station is called the “Chiki” capital of


India, as its famous for the large number of outlets
manufacturing this sweet there. Which one?
LONAVALA

7. Discovered by Hugh Malet in May 1850, it was


developed by Lord Elphinstone. It derives it’s name
from a local word meaning “Mother Forest”. The place
has the famous Lake Charlotte and also it’s monkeys.
MATHERAN- SOME GAVE THE ANSWER AS YERCAUD, WHICH IS
DERIVED FROM TAMIL WORD MEANING "LAKE FOREST"

8. This lake located in Mt.Abu is believed to have


been dug by the nails of the Gods, and gets it’s name
from that legend. Name it.
LAKE NAKKI(POINTS GIVEN FOR NAKKI JEEL ALSO)

9. In 1827 Capt Young a British military officer


explored and developed this place. The place is
believed to have got it’s name from the native shrub
that grew in that vicinity.
MUSSORIE- SOME WRONGLY GAVE THE ANSWER AS KODAIKANAL-
KODAI IS FAMOUS FOR KURINJI FLOWER, BUT THAT HAS
NOTHING TO DO WITH PLACE NAME

10. Often called as the “Scotland of India” due to


it’s similarity with the Scottish highlands. The town
gets it’s name from the name of the local deity,
meaning Superpower of God, who is believed to be
dwelling on the mountain peak that overlooks this
place.
SHILLONG- AGAIN NO POINTS FOR MEGHALAYA. I ASKED THE
NAME OF THE TOWN SPECIFICALLY

Hussain-2
Anagha-2
Shafeek-3
Charan-3
Amlan-5
Paresh-4
Abir-9
Gaurav-9
Ravi Bhatia-3
Shrikant-6
Samrat-6
Nikhil-5
Bala-7

__________________________________________________________
Yahoo! India Matrimony: Find yoThis hill station nested in the
Shivaliks is widely
believed to have taken it’s name from a Tibetan word
meaning “Assembly of King’s ministers”. After the
Anglo Bhutan war came to an end with the Treaty of
Sinchula in 1840, this place was ceded to the British
who developed it as a hill station. Name the place
KALIMPONG-MANY GAVE THE ANSWER AS DARJEELING, WHICH
MEANS "LAND OF THUNDERBOLT"

2. Dharamsala in Himachal Pradesh is located on this


mountain range, which gets it’s name from the meaning
“White Mountain”. These ranges are inhabited by the
Gaddi tribe.
DHAULADHAR-NO POINTS FOR DHAULAGIRI

3. This Indian hill station contains a large number of


Playwin lottery centers, and in fact the Govt of the
state earns a lot of revenue through authorized
gambling. Which place.
GANGTOK- AGAIN NO POINTS FOR SIKKIM, I HAD ASKED THE
NAME OF THE TOWN, NOT THE STATE

4. Legend has it that Lord Hanuman rested here on his


way to fetching the Sanjeevani herb. It also contains
the Pasteur Institute established in 1900 by Sir David
Semple is one of the oldest and produces anti rabies
vaccines. Which place?
KASAULI-LARGE NUMBER OF GUESSES INCLUDED SIMLA, OOTY

5. In 1819 two civil servants of the Madras Govt


Whish and Kindersley, on a trip to Nilgiris, came back
and reported the “discovery of a table land with
European climate”.. It was developed by John Sullivan,
and take’s it’s name from the tribes living in the
hills around it.
KOTAGIRI-AGAIN A LOT OF OOTY ANSWERS, KOTAGIRI WAS
NAMED AFTER THE KOTA TRIBES IN THAT AREA

6. This hill station is called the “Chiki” capital of


India, as its famous for the large number of outlets
manufacturing this sweet there. Which one?
LONAVALA

7. Discovered by Hugh Malet in May 1850, it was


developed by Lord Elphinstone. It derives it’s name
from a local word meaning “Mother Forest”. The place
has the famous Lake Charlotte and also it’s monkeys.
MATHERAN- SOME GAVE THE ANSWER AS YERCAUD, WHICH IS
DERIVED FROM TAMIL WORD MEANING "LAKE FOREST"

8. This lake located in Mt.Abu is believed to have


been dug by the nails of the Gods, and gets it’s name
from that legend. Name it.
LAKE NAKKI(POINTS GIVEN FOR NAKKI JEEL ALSO)

9. In 1827 Capt Young a British military officer


explored and developed this place. The place is
believed to have got it’s name from the native shrub
that grew in that vicinity.
MUSSORIE- SOME WRONGLY GAVE THE ANSWER AS KODAIKANAL-
KODAI IS FAMOUS FOR KURINJI FLOWER, BUT THAT HAS
NOTHING TO DO WITH PLACE NAME

10. Often called as the “Scotland of India” due to


it’s similarity with the Scottish highlands. The town
gets it’s name from the name of the local deity,
meaning Superpower of God, who is believed to be
dwelling on the mountain peak that overlooks this
place.
SHILLONG- AGAIN NO POINTS FOR MEGHALAYA. I ASKED THE
NAME OF THE TOWN SPECIFICALLY

INFO BASE: Name the word or sequence with which a folder cannot be
renamed in WINDOWS-XP. Guess everyone would have got this as a mail
forward!!!
ANS: CON, LPT1-LPT9, COM1-COM9, AUX….i never knew there were
so many, because these are all MS-DOS commands …barb says that it
has been corrected in SP2

1. Fiction. Ensei Tankado wrote a program that creates unbreakable codes


which could not be decrypted even by TRANSLTR, the powerful system
housed by the National Security Agency. How Tankado has named it and how
do we know it better?
ANS: Digital fortress…Disagree with u ratnakar, Dan Brown roxx..

2. This movie was marketed in the US as 'Move it like Mia' because the
original title of the movie was not expected to create any stir as the name used
was unpopular in the US. Name the movie and the director.
ANS: Bend it like Beckham by Gurinder Chadha…many have not mentioned
the director. Got a lot of 'pulp fiction' answers.

3. Which is the longest word that could be typed using only the top row of a
standard typewriter or a computer keyboard?
ANS: 1. TYPEWRITER(10) -word found in dictionary
2. protereotype(12), Proprietory(11), and rupturewort (11)-
words not in dictionary
Thanks Ashwin…

4. It was known as 'Mardi Gras' in the past. How do we know it know? (No
clues attached!!!)
ANS: Saarang the cultural festival of IIT-M; Thought this would be universally
cracked as it was going on when I put the questions, but not satisfactory…

5. CRIC STATS: Which Indian batsman has never been run-out in his entire
test career (should have played minimum of 25 matches)?
ANS: Kapil Dev

6. Started in 1939 by two Stanford engineers, the company has its head quarters
in California. Walt Disney was one of its first big customers, it was involved in
the making of the film 'Fantasia'. In early 2002 it made a merger with another
big corporation. Name the company.
ANS: Hewlett Packard (HP)…See starting word of next question, just a
coincidence, kudos to the top scorer for spotting that

7. HP MANIACS à what is the meaning of the word 'voldemort'. In other


words what
does the word 'voldemort' literally translate to?
ANS: 'Flight of Death'- cracked by many, it's a literal French translation as
mentioned by one of the scorers. Guess it's not there in the book…

8. CONNECT: Kora kagaz


Pinjar
Death of a city
Kagaz te kanwas
The Revenue stamp
ANS: Works by Amrita Pritam

9. Name the Malayali Goddess of education or learning. Good significance out


there. Put funda…
ANS: I have awarded points for 'Manorama' but see what Shafeek says,
"I'm not sure there are any Malayali Goddesses as such distinct from
other Hindus. If the answer you're looking for is "Manorama" (which I've
heard at another quiz), I'd think it is wrong. Manorama means
entertainment in Malayalam. (source: Malayala Manorama. See their
history on the website)"

Please confirm any malyalees around…but I go with Shafeek.

Kudos Anurag singh for somehow relating me to the question…


"Nammakkali (She was the Srinivas Ramanujam's
Goddess-in-cheif, for lack of a better word; And you
are Ramanujam urself)"

10. QUOTE: "Now I can wear high heels", who said this recently and why?
(No part points!!! Hint: Think Hollywood)
Nicole Kidman, after her divorcee with Tom Cruise as she was taller than him.
Also one another friend has put up this question in his quiz…

Quizzing in love
Ramanujam
http://www.ramanujamp.blogspot.com

This company founded in 1833 by Leon Pavin started out as an industrial lime
producer and today is the world's largest cement manufacturing company.
Identify.
Ans:

2) A civil war veteran and Atlanta pharmacist; he stirred up a fragrant, caramel


colored liquid and when it was done, he carried it a few doors down to Jacob's
Pharmacy. Identify the gentleman.
Ans:

3) If HLN is the Hilton Hotels then what is FS?


Ans:

4) This gentleman started his career as a chef in Hotel Majestic Paris but later
joined another industry whose perspective he has changed since. Identify.
Ans:

5) Which bank, founded by Chidambaram Chettyar, commenced operations in


1937 from Karaikudi?
Ans:

6) This product was developed by Roche in 1933. Initially it contained


phenacetin as the active ingredient. But later phenacetin was replaced by
acetaminophen, a proven safe and effective drug that was easier on the stomach
than aspirin. It was brought to India in 1969, thirty-three years after its launch.
Identify.
Ans:

7) Connect Save A Child's Heart, Foundation CURE, Foundation SAHI,


Aragonda Model and 1066.
Ans:

8) One from the Bangalore ET in the Classroom Quiz:


Connect OJSC Ish Mish (a Russian company), which produces bikes namely
Jupiter and Bulleta to something famous/infamous in India.
(Believed to be the best question/answer of the quiz)
Ans:

1) X was declared as the richest young Aussie celebrity. In its 50th anniversary edition, the
Guinness book of world records named X as the highest paid actor/actress for a commercial. X
was given $3.71m for a 4-minute role in a Channel no. 5 advertisement directed by Baz
Luhrmann ? that?s an amazing $928,000 per minute. Who am I talking about?

Ans:

2) ?Only the dead have seen the end of the war?- PLATO, appears at the beginning of which
movie? (It is not the internationalized version of Ashoka)

Ans:

3) Going with the election fever that had spread in the country, what is the science of analyzing
electoral outcomes and predicting them based on voter surveys? It comes from an ancient Greek
word meaning ?pebble?, which the Greeks used as ballots.

Ans:

4) Among the five top-most grosser in animated film history, which is the one exclusively made by
DreamWorks and has grossed $921 million overall?
Ans:

5) The U.S district court, San Francisco has fined which company (MNC) $300 million on the
charges of price-fixing? It was charged of colluding with rivals from 1999 to 2002 to fix prices of
DRAM chips, used in almost every appliance, forcing computer majors such as dell, HP and
APPLE to raise prices to compensate. This was the second largest criminal antitrust fine in
history.

Ans:

6) ?La Noche del 10? or ?The night at 10? aired on Monday nights has which famous celebrity as
its host? He has even interviewed Fidel Castro on the show.

Ans:

7) A simple connect: Connect the movies Charlie and the Chocolate factory, Edward Scissorhand
and Pirates of the Caribbean. Don?t get complicated. It is a simple connect.

Ans:

8) Shefali Chowdhary, a second-generation Bangladeshi girl, plays the role of Parvati Patil in
which recently released Hollywood movie?

Ans:

9) Which novel is considered the first ever graphic novel?

Ans:

10) Who said this and on what occasion (fairly guessable!): ?As a boy, I attended the Bobby
Charlton School with dreams of becoming a professional footballer. I want to give children the
chances I had.?

Ans:

Happy Quizzing!!!

Time for another mixed bag :

1. Name the cricketer who scored a century on his first class debut batting at number eleven
position ?

Ans :

2. 'Konw Why' is the adline of which publication ?


Ans :

3. Who wrote the book ' With a straight bat ' ?

Ans :

4. 'Beauty' is the pet dog of which celebrity actress ?

Ans :

5. In sms lingo, what is 'gl,hf' ?

Ans :

6. what common honour connects Atahualpa, Monet, Martin Luther King, Keats, Chopin etc etc ?

Ans :

7. In history, by what name the thousand year odd period between the fall of the Roman Empire
and discovery of America is known as ?

Ans :

8. Complete the sequence .... Mumbai, Mahim, Parel, Colaba, Mazagaon, Matunga and ?

Ans :

9. If Yudhisthir was born out of Dharma, of whom was Duryodhana born ?

Ans :

10. 'The one above' .... tagline of which car ?

Ans :

1. Here are the answers :

1. A famous sculpture of Michaelangelo inspired Charlton Heston


to be selected for a famous role in a famous Hollywood movie
because of Heston's physical resemblance to the man depicted in
the sculpture. Which film ?

Ans : 'Ten Commandments'. Heston portrayed 'Moses'. The


sculpture is Michaelangelo's 'the statue of moses'.
2. Name the bowler who bowled unchanged in a test innings.

Ans : Mohammed Nazir of Pakistan, 50 overs.

3. 'm-c-c-' fill in the blancs to form a word .... name of a


south american monkey.

Ans : Macaco

4. This place in West Bengal, originally named after a jain


religious head was known as 'sharifabad' during muslim rule. What
is it's the present day name ?

Ans : Burdwan ( Bardhaman ), namesd after 24th Jain Teirtankar

5. By what name we better know 'genfersee' ( local name ) ?

Ans : Lake Geneva

6. 'f-c-n-r-u-' .... fill in the blancs to form a word meaning


'extremely wicked'.

Ans : Facinorous

7. Ritika and Krishen are wife and son of which ace Indian
sportsman ?

Ans : Arjun Atwal

8. What are Rex, Norway, Roof, Sewer types of ?


Ans : Rats

9. In Australia, if you're on a blower, what exactly are you


doing ?

Ans : Talking over telephone

10. 'Your world. Our people' base line of which consulting


company ?

Ans : PricewaterhouseCooper
“SUN rises here “ .. wat s the importance of this phrase? The phrase has
something to do with the picture!. Identify the connection.(
http://image005.mylivepage.com/chunk5/75802/64/1.1.jpg)

2. This image indicates a very famous company!!!. The company’s name has got
the credit of most useful word in the computer era. Identify the company and the
relation with the picture!
http://image005.mylivepage.com/chunk5/75802/64/1.2.jpg)

3. whose tagline is this? “Let your Heart Fly”.

4. This personality owns a famous engineering college in India. And stands 11 th in


a famous list from India. Identify the personality and the college he owns?

5. Identify the importance of this posture…


http://image005.mylivepage.com/chunk5/75802/64/1.5.jpg

6. how is “marshall mathers ” known now?

7. which of the following was not the name of a worm, virus or bug to affect users of
Microsoft's products during the past few years?
(a)CodeRed (b) ILOVEYOU (c) GatesSmasher (d) Melissa(e) MyDoom.

8. When there are two full moons in the same month, what is the second called?

9. Celine Dion - "A new day has come"-- whats the controversy???

10. What is the unit of mouse movement?

11. identify the personality! With the first picture . If you cant, you take the clue in
the second picture!

http://image005.mylivepage.com/chunk5/75802/64/1_11_1.JPG

http://image005.mylivepage.com/chunk5/75802/64/1_11_2.jpg

12. Identify the owner of the voice! And the famous movie which screened the audio?

http://file005.mylivepage.com/chunk5/75802/57/Sound%201.wma
So here goes:

1) Sitter to start with: Priyanka Chopra on the cover page of the


first
issue of this magazine. Which magazine am I talking about?

2) Continuing on the same territory: We all know that Marilyn


Monroe
graced the cover page of the first issue of Playboy Magazine. But there
is
another thing which is very conspicuous by its absence from the
first cover
page of the magazine. What was missing?

3) He once commented "If you dont have a job, start a business".


He is
considered to be a pionner in his field and through his vision the
company
which he has founded has reached the levels it is at presently
today. The
employees of his company are often called "Imagineers". Who am I talking
about?

4) This video game is now part of the classic genre and often being
compared to be the first one which bought "film noir" into the video
gaming
mode. Manufactured and published by Rock Star Entertainment,
which video
game is this?

5) Connect: "Scholar", "Suggest", "Sets", "Transits" and "Reader"

6) As part of the company's hiring policy and post hire training,


all
new recruits of the company and made to watch a tape and listen to the
voice
of the company's founder, in which the founder talks about the
principles of the company and the foundations on which it is based. Name
this company. (Any other clues would be a sitter, but all I can
say
that it is one of the biggies in the global hospitality industry)

7) This agency's founder was very fond of apples. He was so fond of


them that when he opened the first office of this agency he made sure
that a
bowl of apples was placed at the reception area, so that any
visitors
could enjoy munching on them. Now this practice is followed in all the
offices of this agency across the world. Which advertising agency?

8) A famous book "Barbarians at the Gate". Which tobacco company's


story does it tell us?

9) Connect: "Normandie", "Alps", "Region Industrialle",


"Schwarzwald",
"Monaco", "Corsica", "Pyreenees" (apart from being names of places)

10) With which clothing brand in India would you associate Kewal
Kiran &
Co.?

11) Michelin, the famous tyre company, does something very coveted
in
France. To be named and mentioned in this activity is considered to be a
honor and the very best in the industry vie for the award. What
does
Michelin do in France?

12) Sitter: First order delivery for this company was 8 graphic
oscillators to Disney, which Disney used while making the movie
"Fantasia".
It is mentioned in a biography of the founders, that which
founders name
will come first in the name of the company, was decided by a toss of a
coin
between the two founders. Which company am I talking about?

13) This is question is accepted plagiarism: David Ogilvy once


mentioned
that ____________ is the only worthwhile magazine to read in this
world. In
response to this __________ donated a hearty sum of money to
Ogilvy's
old school in Scotland. Which magazine?

14) Applause Entertainment, is a movie production house owned by


which
Indian business group?
15) When this luxury car brand entered the Indian market, it
estimated
that only 20 of this model will be sold in the first year. But, 21
orders
were booked for this car in the first three months, with Delhi
outstripping Mumbai. This forced the company to rethink its estimate
figures. The demand for this car brand influenced many others,
with Audi,
BMW and Posche following suit. Which car brand am I talking about?

16) She has a morbid fascination for tatoos, knives and things
which are
gory. It is said that in her first wedding, she wore a vial which
contained
the blood of her would be husband. Voted as one of the most
beautiful woman
in the world, she has a human side to her nature, which she displays
through
her humanitarian work. Who am I talking about?

17) Some sports to end with (exception from pure business): Which
famous
sport club, some time back ended its long tradition of not having a
sponsor
logo on the player's jersey?

Thats all for now guys. Hit the "Reply" button and send me the answers.

Answers:

1.The biscuits were first thought to have been made by


Australian and New Zealand women for the Australian
and New Zealand soldiers of World War I and were first
called Soldiers' Biscuits.
ANZAC Biscuits

2. Identify the people in the pic

Pizza Hut Brothers- Dan and Frank Carney. (Many had


guessed it as Dick and Mac Mc DOnald)

3. Connect
A food brand endorsed by Drew Barrymore – Baskin
Robbins
"The Open Kettle." Mr. William Rosenberg's , founder
of Dunkin first coffee and donut shop,was called the
"Open Kettle
Michael T Cobler : founder of Togo's (whose complete
name is Togo's Great Sandwiches), a chain of fast-food
sandwich restaurants now owned by Dunkin' Brands Inc.
Baskin-Robbins, Togo's, and Dunkin Donuts comprise
Allied Domecq Quick Service Restaurants, part of
Allied Domecq plc. Allied Domecq was he answer I was
looking for.

4. Name the company behind "Brand new you" campaign,


and the "10/10 Challenge" that promised a 10-pound
weight loss in 10 weeks.
General Mills

5. The three characters were originally designed by


illustrator Vernon Grant and made their debut in 1933.
Their names are derived from a radio ad. The ad said,
"Listen to the fairy song of health, the merry chorus
sung by __________________ as they merrily X, Y and Z
in a bowl of milk. If you've never heard food talking,
now is your chance." Inspired by this ad, Grant drew
three characters and named them brought the
characters to ______ who bought them on the spot.
Opinion varies concerning Y’s occupation, but X is
always portrayed as a baker and Z as a soldier.
Identify X, Y, Z and the brand in question.
Snap, Crackle Pop; Kelloggs
6.
McDonaldland characters Birdie the Early Bird
Grimace, Hamburglar

7. Which company is associated with the foll: Sir


Shakes-A-Lot, the Duke of Doubt and the Wizard of
Fries
Burger King

8. Connect
• Secret recipe of eleven herbs and spices remains one
of the best-kept trade secrets in business. The
original, handwritten recipe is locked securely in a
vault in Louisville KFC
• A character from Treasure island : Long JohnSilver
• Wing Street :chain owned by Pizza hut

The connect I was looking for Yum Brands. Most of you


had answered it as KFC

9. This company ran into trouble by introducing sweets


shaped like flattened snakes, chickens and squirrels
with track marks on their bodies; and was threatened
by New Jersey's Society for the Prevention of Cruelty
to Animals (NJSPCA) to stop the product, saying it
encouraged acts of cruelty. Name the company and the
brand of sweets.

KRAFT - fruit-flavoured Trolli Roadkill Gummi candy

1. Who was the first player to bat through a completed innings in a one-day
international? David Lloyd (Completed innings refers to the full 50 overs here, I
think or something else?) Many of u wrote Grant
Flower…I dunno….maybe it is. Cud anyone verify? Controversial question…legal
advice needed…

2. Who was the top-scorer for India when they were bowled out for 42 by England at
Lord’s in 1974? Eknath Solkar (18) Many of u add – He hit 2 sixes
in this knock.

3. On Boxing Day 1999, which EPL club became the first British side to field an
entirely foreign (non-UK) line-up in a Premier League match against Southampton?
Chelsea

4. Cricket Connect – U Ranchod (Zimbabwe), Ruwan Kalpage (Sri Lanka), Mark


Ealham (England), Neil Johnson (Zimbabwe), Monty Panesar (England), Jacob
Oram (New Zealand). All these
player’s first wicket was Sachin Tendulkar

5. His middle name is John. He was born on March 2, 1977 in Johannesburg,


Transvaal, South Africa. In May 2004, he wrote his name into the record-books
when he became only the fourth batsman to score a century at Lord's on his Test
debut. (The other being - Henry Graham of Australia in 1893, John Hampshire of
England in 1969, and Sourav Ganguly of India in 1996). He was one of the Wisden
Cricketers of the Year for 205. I think enough said of this gentleman, who is “he”?
Andrew Strauss (Expectedly, got many replies as “Kevin Pieterson” due to the SA
born fact)
6. In 1955, which world-champion driver died in an accident during free practice at
Monza? Alberto Ascari

7. Who was the only woman ever to beat American Helen Wills Moody at Wimbledon?
Kitty Godfree

8. Who is awarded the – “Sir Walter Lock Trophy”? Winner of the Kolkata Super
League (Domestic Football)

Most Humorous Answer – “It’s Winner” (Nobody got this one, real toughie!)

9. Glenn Burke is credited with inventing which famous sporting convention? The
High-Five

10. What happened as a result of a a luncheon at the Taplow Club in New York City on
January 16, 1916, held by the department store magnate Rodman Wanamaker,
where several prominent golfers and other leading industry representatives
including Winged foot architect AW Tillinghast and Walter Hagen were present?
The PGA of America

MAIL Q's:

1) Which island nation will host its first ODI and test match in the upcoming series between India and
West Indies?
2) Who won the world billiard championship for women recently?
3) Leander Paes and Mahesh Bhupathi recently created a record in the Davis cup ties, what is the record?

SMS Q's:

1) Which team won the Ranji trophy for ODI recently?


a) Mumbai b) Baroda c) U.P d) Railways
2) Which AUSTRALIAN selector, who held the post for 10 years resigned last week?
a) David boon b) Trevor Hohns c) Doug Walters d) Andrew Hilditch
3) Who was recently appointed as GOODWILL ambassador for UNICEF family?
a) Sachin Tendulkar b) Sania Mirza c) Roger Federer d) Michael Owen
4) Which famous cricketer's midde name is Thomas?
a) Graeme smith b) Michael Vaughan c) Ricky Ponting d) Andrew Flintoff
5) Albert Park is a
a) Opera house in London b) F1 venue c) Cricket ground in Antigua d) US open venue

STRAIGHT DRIVE:
1) These two cricketers are the most capped players for their country with 296 tests between them, they
were born on the same day on the same city, name them?
2) Which venue is scheduled to host the two charity ODI's of the Kashmir 2005 earthquake victims
between India and Pakistan?
3) Name the award bagged by Samresh Jung for being the outstanding sportsman in the commonwealth
games 2006.

QUOTE OF THE WEEK:

"I don't think there is a need for coach in international cricket, every player at this level knows exactly
what to do. The only thing a coach does is to punch worthless data into computer"
Who said this?

SEQUENCE SURPRISE:

Ian Thorpe, Suzie 'o' Neill, Graham Smith and __________.

TOUGHIE:

1) By what name do we know Jyotendra Singhji Vikram Singhji sahib, member of Jadeja dynasty and keen
motor racing enthusiast and former race driver better?
2) With reference to cricket what is TIFWAY 419?
3) Which athletic star of Indian origin had a good communal game winning the bronze medal for women's
200m sprint?

JUMBLED JOY:

Identify the sports car manufacturer in the jumble-IUCTDA.

CLUE 4 U:

"The object on the floor on which one wipes one's feet "was" particular shade of colour "but" this should
have been known before hand"
Identify the sportsperson in the clues.

CARRY OVER:
In the world of Motorsports the car AUDI R 10TI created a world record, what was it?

WHICH VENUE AM I?

1) I was called Maitland formerly. My first test was held on 1984.


2) I am the headquarters of my country's cricket administration body.
3) Tennis court end and south end are the famous ends in my ground.
Identify me.

EXTRAS (Photo Find): (Plz see attached Photos)

1) Which former Indian Badminton player met with a tragic end?


2) Identify the Indian hockey star who is a drag flick specialist.
3) He played just one test for England and got two wickets, that of Sachin and Azhar. He died at the age
of 43. Identify him.
4) Identify the racing star in the photograph.
5) Identify the EPL star who is vying for a place in the England football team.

ANSWERS:

E-mail Q's: 1-St.kitts and Nevis, 2-Chitra magimairaj, 3-Most Davis cup doubles win.

SMS Q's: 1-d, 2-b, 3-c, 4-c, 5-b.

STRAIGHT DRIVE: 1-WAUGH bros, 2-Abu dhabi, 3-David Dixon award.

QUOTE OF THE WEEK: Shane Keith Warne.

SEQUENCE SURPRISE: Alexandra Orlando (Most gold medals-6 in CWG)


TOUGHIE:1-Maharaj Kumar of gondal,2-Grass brought from America and planted in Feroze Shah Kotla
ground for the first India vs England ODI,3-Gerardine Pillai of South Africa.

JUMBLED JOY: DUCATI

CLUE 4 U: Mathew Prior.

CARRY OVER: First Diesel powered car to win a motor car race.

WHICH VENUE AM I: SSC-Sinhalese sports club, Colombo

EXTRAS (Photo Find):1-Syed Modi, 2-Jugraj Singh, 3-Neil Williams, 4-James Beattie, 5-Loris Capirossi.

1. ) The Business Quiz:

The Business quiz should be of special interest to all B-school


quiz
enthusiasts, with lots of visuals and audio questions, and
spanning the
entire gamut of biz quiz questions-from sports to showbiz, from
brands to
celebrities.

There are many attractive prizes, which will be announced shortly


by the
NITR people.

Venue: AV Hall, NIT Rourkela


Date 04-02-2006
Time: 9:30 A.M.

2) The General Quiz


The General Quiz promises to be a crowd puller and this is where
serious
quizzers can show the rest of the junta just where they have an
edge over
the rest. This also will be a content rich quiz, with as much
importance
given to entertainment as to information

Venue: AV Hall, NIT Rourkela


Date 05-02-2006
Time: 9:30 A.M.

3) The Entertainment Quiz

If you think you're a dude when it comes to knowing about


Entertainment,
this quiz just might make you think again. I shall say no more
about the
quiz itself, but let the true quizzers separate themselves from
the kids on
the battlefield itself.

Venue: AV Hall, NIT Rourkela


Date 05-02-2006
Time: 9:30 A.M.

Read on for some teaser questions from all three quizzes.

Business

Q. Which Indian firm which traces its origins to a company


founded by
Walchand Hirachand in 1940 crossed the $1 bn revenue mark in the
last
financial year?

Q. Which bank financed the Louisiana Purchase, Phileas Fogg's


Journey around
the world in 80 days, Napolean's war effort amongst other things
and
collapsed on February 26, 1995 thanks to the activities of one
rogue trader
who lost $1.4 billion by gambling on the SIMEX?
Q. Nihon Keizai Shimbun is a Japanese corporation that publishes
financial,
business and industry news. What is it popularly known as?

Q. The World Bank, the Government of India and representatives of


Indian
industry formed it as a development finance institution to
provide
medium-term and long-term project financing to Indian businesses
in
1955.This year it acquired Investitsionno- Kreditny Bank (IKB), a
Russia
bank with about US$4mn in assets, headoffice in Balabanovo in the
Kaluga
region, and with a branch in Moscow. What are we talking about?

Q. The English king Offa decreed that two hundred and forty coins
were to be
struck from a pound of silver. By doing this what did he
introduce?

General

Q. In 1895, he drafted the Electricity Act of Britain. In 1899, a


dynamo
manufactured by him generated electric power for the first time
in India at
Calcutta. Who?

Q. When it was discovered, it was named "Georgium Sidium", in


honour of King
George III of England. For many years it was known as the
"Georgian". Only
in 1850, it was named after a Roman God, and the name has
remained as what
we know it as. What are we talking about?

Q. The basic criteria for a certain movie were along the lines
that - A
Bengali played a Punjabi farmer formerly in the army. A Keralite
played a
Bengali footballer. A Brahmin from Meerut played a Harijan from
Tamil Nadu.
An Iranian Muslim played a Maharashtrian tamasha artiste. An
Urdu speaking
Muslim played a Hindi pracharak Varanasi Brahmin. A Muslim played
a
Christian girl of Portuguese origin. A Brahmin played a Muslim
Urdu poet
from Bihar. Which movie?

Q. How do we better know the Sapthagiri Hills?

Q. In sandlot baseball, when it was required that the two teams


determine
who would bat first, they resorted to a curious practice that
gave rise to a
phrase. Which phrase and how did it come about?

Entertainment

Q. In Amelie whose painting does Audrey Tautou's neighbour


attempt painting
throughout the movie?

Q. What do Jug Suraiya and Neelabh create for the Times Of India?

Q. How do you better know the author called L.M Olehewitz?

Q. Built between Thailand and Burma it was 415km long (about 303
kms in
Thailand and about 112 kms in Burma) and passed through the
Three Pagoda
Pass in Sangkhlaburi District, the most northern part of
Kanchanaburi
province. Construction was began on September 16, 1942 at Nong
Pladuk,
Thailand by approximately 30,000 from England, Australia, Holland
and
America and more than 200,000 impressed labourers from India,
China,Indonesia, Malaysia, Singapore, Burma and Thailand. Of
these, more
than 16,000 workers and 100,000 impressed labourers died of many
diseases,
due to starvation and lack of medical equipment. It is said that
the first
survey by the Japanese engineers predicted that it would take at
least five
years to finish this railway line, but the Japanese army forced
the
completion in only sixteen months. Thus it was completed on 25
December
1943. What?

Q. He was the first Western explorer to enter the sacred city of


Mecca. He
spoke twenty languages, almost discovered the source of the
Nile, and wrote
the first Western translation of the Arabian Nights. Who?

Q. Before the 1966 World Cup, they wanted to meet the Brazilian
football
team. They wrote to the Brazilian Federation for permission.
When it was
granted, they went to the hotel where the team was staying.
However the
manager disallowed them saying that the team might try to copy
their hair
style and that this might lead to indiscipline in future. Who
were 'they'?

Q. Which great Wimbledon Champion was a World Men's Table Tennis


title
winner and once also appeared for the audition of the Fred
Astaire starring
movie "Top Hat" ?

Q. What piece of art resulted as when painter Mati Klarwein met


Timothy
Leary in Tangiers?

Q. An English version of the movie is to be made under the


direction of
Mira Nair under the tentative title of Gangsta, M. D.
starring Chris
Tucker. Which flick?

This web based mail service has HTML in it. It was initially backed by Draper Fisher
Jurvetson & was later sold to another Giant. In 1997, it reported more than 8.5 Million
subscribers. What am I talking about?
2. Naomi Klein in her book No Logo had criticized this corporation so much that it became
one of the first publications to receive the first feedback from that corporation. Which
corporate am I talking about?
3. Lara Croft, Kermit the Frog, Britney Spears, Buffy the Vampire Slayer, Bart & Lisa
Simpson, Austin Powers, Backstreet Boys & …… Now what is this all about?
4. In 1974, Steve Prefontaine became the first athlete to have done this. What is it that he
did?
5. Xserve, Socks, Remote, Dock, Sherlock, Stickies – All products of which corporation?
6. Apart from his very famous ventures, he also helped in the establishment of these
companies: NexGen, Excite, Juniper, Cerent, Corvis. He is also on the board of QWEST
Communications, Asera, Siara Systems. Who is this?
7. Connect: F.R.I.E.N.D.S, Baywatch, Only Fools and Horses, The Rebel Billionaire to a
very famous business personality.
8. If JSP is by Sun Microsystems, ASP is from ?
9. Who or What owns more stocks than any other Stockholder in the largest Co.on the
planet, Exxon Mobil?
10. Connect Sears Roebuck & Co., F.W.Woolworth Co., May Department Stores Co.,
Gimbel Brothers Inc., R.H.Macy & Co.
Answers will be posted this Saturday......

) Who said this: ?Until a few years ago, I thought I would be a go-kart mechanic?.
Hint: F1

Ans:
2) In which Olympics was the official Olympic flag was first flown?

Ans:

3) Juan Pablo Montoya injured his shoulder playing a particular game in 2005 and had to miss 2
races. Name the game?

Ans:

4) Which is now the biggest-valued sports brand in terms of sponsorship revenue? It is now
valued at $27.12 million, displacing Juventus (Italian Club) to second place at $22.2 million.

Ans:

5) Only 2 players have won the French Open in the very first attempt. Rafael Nadal was one of
them. Who is the other?

Ans:

6) Who was the first tennis player, man or woman, to rise from outside the top 100 to rank 2nd in
the same year.

Ans:

7) Ronaldo de Assis Moreira is the birth name of which famous footballer?

Ans:

8) What was common to F1 winner Renault, WRC winner Citron and MotoGp winner Yamaha? A
simple connect.

Ans:

9) How is Shiva Keshavan unique to Indian sport? Hint: Next winter Olympics are in Turin.

Ans:

10) ?China was poor back then, so we loved Tennis because it gave us a chance to wear cool
clothes? ? whose quote is this?

Ans:

11) Who became the sixth person to score a century in his 100th test match and became the first
person to score a century in both the innings of his 100th test match. (A real sitter?)
Ans:

12) Who became the first cricketer in 24 years to win the BBC sports personality award?

Ans:

13) Who was voted among the 50 most beautiful people in the world in 1999, the same year he
was the world No 1 in Tennis? A few months later, he launched his owner cologne and in 2001,
he acted in the Spanish film Torrente II. He loves water and has a tattoo of a dolphin on his bicep.
He is a good friend of the golfer Sergio Garcia.

Ans:

14) Which tennis player has got a new name ?Mahaviro? after completing a week of service at a
Buddhist temple? ?I might lose a match, but I love to win the crowd? is one of his famous quotes.
His nicknames are ?Ball? and ?Superball?.

Ans:

15) Allan Border (27), Steve Waugh (23) ?.. Give me the next person in the list.

Ans:

16) Camille Nevieve is the girlfriend of which now famous Tennis player? He became famous in
the Australian Open.

Ans:

17) Who holds the world record of the maximum test match sixes and how many?

Ans:

18) 212-ranked Australian Mark Edmondson, in 1976, was the last person to do what? Hint: The
guy from Q16 almost did it!

Ans:

19) Which F1 team is being named as Midland, having been taken over by Russian Businessman
Alex Shnaider?

Ans:

20) In the year 2000, which famous world personality, not related to sports, was named an ?
Honorary Globetrotter?? (He is no more, having passed away last year!)
Ans:

Happy Quizzing! Want atleast one 20 pointer for sure!

Varun Reddy Sevva


3rd year, NITK

1) Which spice company is called mahashiyan di hatti-MDH


2) What is "c" in CK Prahlad i)chennaii)coimbatore iii)chikmanglur-ii
3) According to Visa Asia pacific commercial survey which country ranked below
china in the standings-japan
4)who was the assistant of Christan Dior in ........(forgot) i)ralph lauren ii) versace
iii)ysl
5)which artist was friend of luca picholi who created double entry system-da vinci
6)which women's clothing item derives name from a place where nuclear tests
were conducted i)braii)pantyiii)bikini-bikini
7)which well known book updated every year is from HIT entertainment-guiness
8)to releive rheumatic arthiritis in chidren what was invented i)sauna ii) jacuzzi iii)
merrygoround-jacuzzi
9)forgot
10) which american rap artist has a clothing line called shady ltd-eminem
11) some question related to tropicana
12) which ex nasscom chief worked with shyam benegal in manthan-devang
mehta
13) who funds BBC world service programs-british govt
14) forgot
15)first male model for lux-paul newman
16)which day invented by Archies i)mother's day ii)father;s day iii) friendship day
iv) valentine;s day-iii
17) what is the limit of amt in single money order i)1000 ii)2500 iii)5000 iv)10000-
iii
18)what contributes 14% of us budget i)health ii)defence iii)education-i
19)czech and irish are max consumers of which drink i)milk ii)beer iii) water-ii
20) recently which company bagged BCCI television rights-nimbus
21) some peter drucker saying to be completed
22)whose kick formed adidas's ad campaign i)ronaldinho ii)baichung iii) zinedine
zidane iv)beckham-iv
23) alfred grace thomson manufactures what brand of cricket balls-kookabura
24) iso 14001 is for what parameters-Environment
25)which is the highest selling video game in 2005-PS
26) forgot
27)forgot
28) what is the code name for paris airport-CDG after charles de gaulle
29)loacl ahmedabad question
30) spell allen solly

1) He has completed a one year under graduate programme in economics from Harvard and an
M.Phil course in development studies from Cambridge University. He was briefly employed at
management guru Michael Porter's company Monitor. Who?

Ans:

<!--[if !supportEmptyParas]--> <!--[endif]-->

2) He won the Golden Bear at the 1967 Berlin Film Festival for his documentary titled Through
the Eyes of a Painter. A documentary film he made in 1972 titled Calcutta Unedited was screened
last year at the Oberhausen International Short Film Festival in Germany. Who are we talking
about?

Ans:

<!--[if !supportEmptyParas]--> <!--[endif]-->

3) This Indian volleyball player died in an accident in Italy- where he was quite a star- in the
1980s. His family came back into the limelight in 2002 after one of their bahus did something
major. Name this sportsman and the bahu.

Ans:

<!--[if !supportEmptyParas]--> <!--[endif]-->

4) The Mosconi Cup is the major championship in this particular indoor sport. The name of this
form of the sports is also a gross slang term in India for an unmentionable activity. J Which sport?

Ans:

<!--[if !supportEmptyParas]--> <!--[endif]-->

5) This language is spoken in Maharashtra, Karnataka and Kerala and is the official language of
yet another state in the region. It has no script of its own and is generally written in the local script
wherever it is spoken. Identify the tongue.
Ans:

<!--[if !supportEmptyParas]--> <!--[endif]-->

6) During Wimbledon what are stored at a precise temperature of 68 degree Fahrenheit to keep
them in perfect condition?

Ans:

<!--[if !supportEmptyParas]--> <!--[endif]-->

7) A very nice question: Korea was originally spelt with a `c' and there has been a recent move to
revert to this, why was it earlier changed to K?

Ans:

<!--[if !supportEmptyParas]--> <!--[endif]-->

8) By what common name are the large bombs developed in the early years of World War II by
the Royal Air Force known? They weighed around 4000 pounds and were carried by Wellington
and Lancaster bombers on raids into Germany.

Ans:

<!--[if !supportEmptyParas]--> <!--[endif]-->

Answers, suggestions and feedback to: bangquizzers@yahoo.com

<!--[if !supportEmptyParas]--> <!--[endif]-->

P.S: All questions are from quizzes attended by Team QuizKrieg or from public archives

Você também pode gostar